[algogeeks] Re: MS

2011-07-02 Thread SVIX
read everything once insert them in a dictionary of type char,
listint where for each character key, u record the indices (since u
record from left to right, they're gonna be in ascending order)

then, for any character that's asked for, u can get to the list in
O(1) time and can get the min distance...

On Jun 30, 12:40 pm, bittu shashank7andr...@gmail.com wrote:
 lets take the example {1,2, 10, 2, 3, 5, 1, 1,2,3,5,2} clearly min
 distance is 1 between a=2  b=5
 take variables current=-1  min=INT_Max
 so we start from last element say its index is last which is array
 size -1  check it with a  b if its equal to any of these then check
 current !=-  a[current]!=a[last] if its true then compare min with
 min  current-last  update the min e.g min=minimum(min,current-last)
 set current=n; repeat until we run out of loop

 so function looks like
 minDistance (int a[], int n, int b, int c)
 {

 int ret = INT_MAX, cur rent= -1;

 while(n--)
 {

 if(a[n] == b || a[n] == c) {

 if(cur rent!= -1  a[current] != a[n]) {

 min = minimum(min, current - n);}

 current = n//upodate current

 }

 Thanks
 Shashank Mani
 CSE, BIT Mesra

-- 
You received this message because you are subscribed to the Google Groups 
Algorithm Geeks group.
To post to this group, send email to algogeeks@googlegroups.com.
To unsubscribe from this group, send email to 
algogeeks+unsubscr...@googlegroups.com.
For more options, visit this group at 
http://groups.google.com/group/algogeeks?hl=en.



Re: [algogeeks] Re: Sum to K problem

2011-07-02 Thread Navneet Gupta
Wrote the code for same.


#includeiostream
using namespace std;

int max(int a,int b)
{
if(ab)
return a;
else
return b;
}

int main()
{
int n,k;
cout\nEnter the count of numbers :;
cinn;

//Set of Elements
cout\nEnter the elements :;
int *A;
A = new int[n];
for(int i = 0; in; i++)
cinA[i];

//Input Sum Value
cout\nEnter the value of k :;
cink;

//Matrix for holding boolean values for subproblems (max subproblems 
upto nk)
int **M;
M = (int **)new int[n];
for(int i=0; in; i++)
M[i] = new int[k+1];

//Populate all the values to false
for(int i=0; in; i++)
for(int j=0; j=k; j++)
M[i][j] = 0;

for(int i=0; in; i++)
M[i][0] = true;

for(int i=1; in; i++)
for(int j=0; j=k; j++)
if(j-A[i]=0)
{
M[i][j] = max(M[i-1][j], M[i-1][j-A[i]]);
}

if(M[n-1][k] == 1)
cout\nPossible Subset present;
else
cout\nPossible Subset not present;

cin.get();
return 0;
}


On Fri, Jun 24, 2011 at 11:42 PM, ross jagadish1...@gmail.com wrote:
 This is the subset sum problem which is NP,.

 The DP is as follows,

 M[i,j] = 1 , if a subset of first i elements has a sum = j.
 else 0,
 The recurrence is,
 M[i,j] = max(M[i-1,j] , M[i-1,j-Ai]) where Ai is the ith element.

 You can maintain back pointers to keep track of previous elements so
 that
 the solution can be reconstructed from the DP.

 Once this matrix is populated till sum=k, Then, the column
 corresponding to sum=k, gives
 the answer. complexity o(nk).



 On Jun 20, 6:38 pm, Harshal hc4...@gmail.com wrote:
 The problem is NP. Complexity using DP will be O(nk), where n is number of
 elements and k is required sum.

 S[0]=true; //choose no element
 S[1...k] = false;
 for each number i in your input
  for s = k downto i
         if ( S[s - i] == true )
             S[s] = true;

 S[s] = true indicates a sum of i can be obtained from a subset of the set.
 To get the elements, we can make S[s] contain the list of numbers that
 contain the sum.
 On Mon, Jun 20, 2011 at 6:53 PM, Navneet Gupta navneetn...@gmail.comwrote:









  Ideally, yes it can. Though i would be happy even if someone gives a
  good answer for non-negative values.

  On Mon, Jun 20, 2011 at 6:28 PM, Akshata Sharma
  akshatasharm...@gmail.com wrote:
   @Navneet: does the set contains negative elements also?

   On Mon, Jun 20, 2011 at 12:26 PM, pacific :-) pacific4...@gmail.com
  wrote:

   @vaibhav : Please note that more than two numbers can sum upto k.

   On Mon, Jun 20, 2011 at 12:21 PM, vaibhav shukla 
  vaibhav200...@gmail.com
   wrote:

   sort the array using merge sort : order nlogn
   take the first element,subtract it with 'k' , then search the result
   using binary search in rest of the array : order nlogn
   hence u get two elements which sum up to K in order nlogn

   On Mon, Jun 20, 2011 at 12:14 PM, Navneet Gupta navneetn...@gmail.com

   wrote:

   Right, in the worst case, complexity with be O(2^N).
   So what are the possible optimizations here? Would building
  pre-computed
   data structures with intermediate sum values help in finding such
  pairs in
   less complexity? I think then we can apply dynamic programming to find
  such
   pairs.

   On Mon, Jun 20, 2011 at 12:09 PM, oppilas . 
  jatka.oppimi...@gmail.com
   wrote:

   I think its a NP problem. The solution complexity can go up O(2^N) in
   worst case.

   On Mon, Jun 20, 2011 at 11:55 AM, Navneet Gupta 
  navneetn...@gmail.com
   wrote:

   Given a set of integers , find a set of numbers such that they sum
  to
   a given number k .
   Notice the set of numbers can have 2 or more than 2 numbers.

   --Navneet

   --
   You received this message because you are subscribed to the Google
   Groups Algorithm Geeks group.
   To post to this group, send email to algogeeks@googlegroups.com.
   To unsubscribe from this group, send email to
   algogeeks+unsubscr...@googlegroups.com.
   For more options, visit this group at
  http://groups.google.com/group/algogeeks?hl=en.

   --
   You received this message because you are subscribed to the Google
   Groups Algorithm Geeks group.
   To post to this group, send email to algogeeks@googlegroups.com.
   To unsubscribe from this group, send email to
   algogeeks+unsubscr...@googlegroups.com.
   For more options, visit this group at
  http://groups.google.com/group/algogeeks?hl=en.

   --
   --Navneet

   --
   You received this message because you are subscribed to the Google
   Groups Algorithm Geeks group.
   To post to this group, send email to algogeeks@googlegroups.com.
   To unsubscribe from this group, send email to
   

[algogeeks] help..

2011-07-02 Thread cegprakash
the length of the rope is l units.
I can only cut any rope into two halves.

for example if the length of the rope is 8 and we need a length of
rope 6

we first cut into two halves and we get 4, 4
now we cut any of the half again and we get 4,2,2

now we can merge 4 and 2 and form a rope of length 6.

in this example we need a minimum of 2 cuts to get the length of rope
6 from 8

assume that l is always a power of 2 and we need always a even length
of rope from it how to find the number of minimum cuts needed to get
the new rope?.

-- 
You received this message because you are subscribed to the Google Groups 
Algorithm Geeks group.
To post to this group, send email to algogeeks@googlegroups.com.
To unsubscribe from this group, send email to 
algogeeks+unsubscr...@googlegroups.com.
For more options, visit this group at 
http://groups.google.com/group/algogeeks?hl=en.



Re: [algogeeks] help..

2011-07-02 Thread Shalini Sah
i guess the no. of 1s in the binary representation of the number is the
answer..for 6 its 2...

On Sat, Jul 2, 2011 at 1:32 PM, cegprakash cegprak...@gmail.com wrote:

 the length of the rope is l units.
 I can only cut any rope into two halves.

 for example if the length of the rope is 8 and we need a length of
 rope 6

 we first cut into two halves and we get 4, 4
 now we cut any of the half again and we get 4,2,2

 now we can merge 4 and 2 and form a rope of length 6.

 in this example we need a minimum of 2 cuts to get the length of rope
 6 from 8

 assume that l is always a power of 2 and we need always a even length
 of rope from it how to find the number of minimum cuts needed to get
 the new rope?.

 --
 You received this message because you are subscribed to the Google Groups
 Algorithm Geeks group.
 To post to this group, send email to algogeeks@googlegroups.com.
 To unsubscribe from this group, send email to
 algogeeks+unsubscr...@googlegroups.com.
 For more options, visit this group at
 http://groups.google.com/group/algogeeks?hl=en.



-- 
You received this message because you are subscribed to the Google Groups 
Algorithm Geeks group.
To post to this group, send email to algogeeks@googlegroups.com.
To unsubscribe from this group, send email to 
algogeeks+unsubscr...@googlegroups.com.
For more options, visit this group at 
http://groups.google.com/group/algogeeks?hl=en.



[algogeeks] Re: conditional compilation

2011-07-02 Thread himanshu kansal
Ya its cleard nw.. Thnx..:)

On 7/2/11, Dumanshu duman...@gmail.com wrote:
 In ur second code change int i=2 and run it. It would still print bye.
 check the result of preprocessor using -E flag.

 On Jun 30, 4:46 pm, himanshu kansal himanshukansal...@gmail.com
 wrote:
 1 more thngif its possible to eval. variable expression lyk abv...
 thn...
 int i=2;
 #if i==2
 printf(hi);
 #else
 printf(bye);     //prints byei think it shld print hi...
 #endif

 even this code prints bye.

 int i=3;
 #if i==2
 printf(hi);
 #else
 printf(bye);
 #endif

 On Thu, Jun 30, 2011 at 5:13 PM, himanshu kansal 









 himanshukansal...@gmail.com wrote:
  somewhere i read dt conditional compilation such as #if and #elif etc
  cn only use constant expressions and previously defined macrosthey
  cant use variables cz preprocessng is done b4 compilation...
  bt whn i try it on gcc and vc both r compiling it with variables abs.
  fyneven disabling extensions dint work
  i.e int i;
  #if i+1
  printf(hi);     vl print hi...no error occurs
  #endif

  thn i read it on net dt it cn eval. varables also
  nw m confused who is ryt...
  whtr it cn eval var also or nt...
  nd if it cn thn hws it possible dt preprocessor knows d val. of var.
  b4 compiling d prog...

 --

       Regards
 Himanshu Kansal
   Msc Comp. sc.
 (University of Delhi)

 --
 You received this message because you are subscribed to the Google Groups
 Algorithm Geeks group.
 To post to this group, send email to algogeeks@googlegroups.com.
 To unsubscribe from this group, send email to
 algogeeks+unsubscr...@googlegroups.com.
 For more options, visit this group at
 http://groups.google.com/group/algogeeks?hl=en.



-- 
Sent from my mobile device


  Regards
Himanshu Kansal
  Msc Comp. sc.
(University of Delhi)

-- 
You received this message because you are subscribed to the Google Groups 
Algorithm Geeks group.
To post to this group, send email to algogeeks@googlegroups.com.
To unsubscribe from this group, send email to 
algogeeks+unsubscr...@googlegroups.com.
For more options, visit this group at 
http://groups.google.com/group/algogeeks?hl=en.



[algogeeks] Re: help..

2011-07-02 Thread cegprakash
that will not work.

for example we need a rope of length 4 from a rope of length 16

we need 2 cuts

16== 8 + 8 == 8+ 4+ 4

-- 
You received this message because you are subscribed to the Google Groups 
Algorithm Geeks group.
To post to this group, send email to algogeeks@googlegroups.com.
To unsubscribe from this group, send email to 
algogeeks+unsubscr...@googlegroups.com.
For more options, visit this group at 
http://groups.google.com/group/algogeeks?hl=en.



Re: [algogeeks] help..

2011-07-02 Thread keyan karthi
yup :)

On Sat, Jul 2, 2011 at 1:38 PM, Shalini Sah shalinisah.luv4cod...@gmail.com
 wrote:

 i guess the no. of 1s in the binary representation of the number is the
 answer..for 6 its 2...


 On Sat, Jul 2, 2011 at 1:32 PM, cegprakash cegprak...@gmail.com wrote:

 the length of the rope is l units.
 I can only cut any rope into two halves.

 for example if the length of the rope is 8 and we need a length of
 rope 6

 we first cut into two halves and we get 4, 4
 now we cut any of the half again and we get 4,2,2

 now we can merge 4 and 2 and form a rope of length 6.

 in this example we need a minimum of 2 cuts to get the length of rope
 6 from 8

 assume that l is always a power of 2 and we need always a even length
 of rope from it how to find the number of minimum cuts needed to get
 the new rope?.

 --
 You received this message because you are subscribed to the Google Groups
 Algorithm Geeks group.
 To post to this group, send email to algogeeks@googlegroups.com.
 To unsubscribe from this group, send email to
 algogeeks+unsubscr...@googlegroups.com.
 For more options, visit this group at
 http://groups.google.com/group/algogeeks?hl=en.


  --
 You received this message because you are subscribed to the Google Groups
 Algorithm Geeks group.
 To post to this group, send email to algogeeks@googlegroups.com.
 To unsubscribe from this group, send email to
 algogeeks+unsubscr...@googlegroups.com.
 For more options, visit this group at
 http://groups.google.com/group/algogeeks?hl=en.


-- 
You received this message because you are subscribed to the Google Groups 
Algorithm Geeks group.
To post to this group, send email to algogeeks@googlegroups.com.
To unsubscribe from this group, send email to 
algogeeks+unsubscr...@googlegroups.com.
For more options, visit this group at 
http://groups.google.com/group/algogeeks?hl=en.



[algogeeks] Re: help..

2011-07-02 Thread cegprakash
nope

On Jul 2, 1:14 pm, keyan karthi keyankarthi1...@gmail.com wrote:
 yup :)

 On Sat, Jul 2, 2011 at 1:38 PM, Shalini Sah shalinisah.luv4cod...@gmail.com

  wrote:
  i guess the no. of 1s in the binary representation of the number is the
  answer..for 6 its 2...

  On Sat, Jul 2, 2011 at 1:32 PM, cegprakash cegprak...@gmail.com wrote:

  the length of the rope is l units.
  I can only cut any rope into two halves.

  for example if the length of the rope is 8 and we need a length of
  rope 6

  we first cut into two halves and we get 4, 4
  now we cut any of the half again and we get 4,2,2

  now we can merge 4 and 2 and form a rope of length 6.

  in this example we need a minimum of 2 cuts to get the length of rope
  6 from 8

  assume that l is always a power of 2 and we need always a even length
  of rope from it how to find the number of minimum cuts needed to get
  the new rope?.

  --
  You received this message because you are subscribed to the Google Groups
  Algorithm Geeks group.
  To post to this group, send email to algogeeks@googlegroups.com.
  To unsubscribe from this group, send email to
  algogeeks+unsubscr...@googlegroups.com.
  For more options, visit this group at
 http://groups.google.com/group/algogeeks?hl=en.

   --
  You received this message because you are subscribed to the Google Groups
  Algorithm Geeks group.
  To post to this group, send email to algogeeks@googlegroups.com.
  To unsubscribe from this group, send email to
  algogeeks+unsubscr...@googlegroups.com.
  For more options, visit this group at
 http://groups.google.com/group/algogeeks?hl=en.

-- 
You received this message because you are subscribed to the Google Groups 
Algorithm Geeks group.
To post to this group, send email to algogeeks@googlegroups.com.
To unsubscribe from this group, send email to 
algogeeks+unsubscr...@googlegroups.com.
For more options, visit this group at 
http://groups.google.com/group/algogeeks?hl=en.



Re: [algogeeks] Re: help..

2011-07-02 Thread varun pahwa
k - rope of desired length.
l - rope of given length
m = 2;
while(k % m)
m *= 2;
ans :: (log2(l) - log2(m) + 1).
ex.
k = 6,l = 8
so initially m = 2;
after 1st iteration m = 4;
then break;
so min = log2(8) - log2(4) + 1  = 3 -2 + 1 = 2.


On Sat, Jul 2, 2011 at 1:16 AM, cegprakash cegprak...@gmail.com wrote:

 nope

 On Jul 2, 1:14 pm, keyan karthi keyankarthi1...@gmail.com wrote:
  yup :)
 
  On Sat, Jul 2, 2011 at 1:38 PM, Shalini Sah 
 shalinisah.luv4cod...@gmail.com
 
   wrote:
   i guess the no. of 1s in the binary representation of the number is the
   answer..for 6 its 2...
 
   On Sat, Jul 2, 2011 at 1:32 PM, cegprakash cegprak...@gmail.com
 wrote:
 
   the length of the rope is l units.
   I can only cut any rope into two halves.
 
   for example if the length of the rope is 8 and we need a length of
   rope 6
 
   we first cut into two halves and we get 4, 4
   now we cut any of the half again and we get 4,2,2
 
   now we can merge 4 and 2 and form a rope of length 6.
 
   in this example we need a minimum of 2 cuts to get the length of rope
   6 from 8
 
   assume that l is always a power of 2 and we need always a even length
   of rope from it how to find the number of minimum cuts needed to get
   the new rope?.
 
   --
   You received this message because you are subscribed to the Google
 Groups
   Algorithm Geeks group.
   To post to this group, send email to algogeeks@googlegroups.com.
   To unsubscribe from this group, send email to
   algogeeks+unsubscr...@googlegroups.com.
   For more options, visit this group at
  http://groups.google.com/group/algogeeks?hl=en.
 
--
   You received this message because you are subscribed to the Google
 Groups
   Algorithm Geeks group.
   To post to this group, send email to algogeeks@googlegroups.com.
   To unsubscribe from this group, send email to
   algogeeks+unsubscr...@googlegroups.com.
   For more options, visit this group at
  http://groups.google.com/group/algogeeks?hl=en.

 --
 You received this message because you are subscribed to the Google Groups
 Algorithm Geeks group.
 To post to this group, send email to algogeeks@googlegroups.com.
 To unsubscribe from this group, send email to
 algogeeks+unsubscr...@googlegroups.com.
 For more options, visit this group at
 http://groups.google.com/group/algogeeks?hl=en.




-- 
Varun Pahwa
B.Tech (IT)
7th Sem.
Indian Institute of Information Technology Allahabad.
Ph : 09793899112 ,08011820777
Official Email :: rit2008...@iiita.ac.in
Another Email :: varunpahwa.ii...@gmail.com

People who fail to plan are those who plan to fail.

-- 
You received this message because you are subscribed to the Google Groups 
Algorithm Geeks group.
To post to this group, send email to algogeeks@googlegroups.com.
To unsubscribe from this group, send email to 
algogeeks+unsubscr...@googlegroups.com.
For more options, visit this group at 
http://groups.google.com/group/algogeeks?hl=en.



Re: [algogeeks] Re: help..

2011-07-02 Thread sunny agrawal
xor the length of the rope with the required length and difference between
the indexes of first set and last set bit *may* be the answer !!

On Sat, Jul 2, 2011 at 1:46 PM, cegprakash cegprak...@gmail.com wrote:

 nope

 On Jul 2, 1:14 pm, keyan karthi keyankarthi1...@gmail.com wrote:
  yup :)
 
  On Sat, Jul 2, 2011 at 1:38 PM, Shalini Sah 
 shalinisah.luv4cod...@gmail.com
 
   wrote:
   i guess the no. of 1s in the binary representation of the number is the
   answer..for 6 its 2...
 
   On Sat, Jul 2, 2011 at 1:32 PM, cegprakash cegprak...@gmail.com
 wrote:
 
   the length of the rope is l units.
   I can only cut any rope into two halves.
 
   for example if the length of the rope is 8 and we need a length of
   rope 6
 
   we first cut into two halves and we get 4, 4
   now we cut any of the half again and we get 4,2,2
 
   now we can merge 4 and 2 and form a rope of length 6.
 
   in this example we need a minimum of 2 cuts to get the length of rope
   6 from 8
 
   assume that l is always a power of 2 and we need always a even length
   of rope from it how to find the number of minimum cuts needed to get
   the new rope?.
 
   --
   You received this message because you are subscribed to the Google
 Groups
   Algorithm Geeks group.
   To post to this group, send email to algogeeks@googlegroups.com.
   To unsubscribe from this group, send email to
   algogeeks+unsubscr...@googlegroups.com.
   For more options, visit this group at
  http://groups.google.com/group/algogeeks?hl=en.
 
--
   You received this message because you are subscribed to the Google
 Groups
   Algorithm Geeks group.
   To post to this group, send email to algogeeks@googlegroups.com.
   To unsubscribe from this group, send email to
   algogeeks+unsubscr...@googlegroups.com.
   For more options, visit this group at
  http://groups.google.com/group/algogeeks?hl=en.

 --
 You received this message because you are subscribed to the Google Groups
 Algorithm Geeks group.
 To post to this group, send email to algogeeks@googlegroups.com.
 To unsubscribe from this group, send email to
 algogeeks+unsubscr...@googlegroups.com.
 For more options, visit this group at
 http://groups.google.com/group/algogeeks?hl=en.




-- 
Sunny Aggrawal
B-Tech IV year,CSI
Indian Institute Of Technology,Roorkee

-- 
You received this message because you are subscribed to the Google Groups 
Algorithm Geeks group.
To post to this group, send email to algogeeks@googlegroups.com.
To unsubscribe from this group, send email to 
algogeeks+unsubscr...@googlegroups.com.
For more options, visit this group at 
http://groups.google.com/group/algogeeks?hl=en.



[algogeeks] Re: help..

2011-07-02 Thread cegprakash
k is an even number and m=2 in your code. k%2 is always 0. your while
loop does nothing.

On Jul 2, 1:26 pm, varun pahwa varunpahwa2...@gmail.com wrote:
 k - rope of desired length.
 l - rope of given length
 m = 2;
 while(k % m)
 m *= 2;
 ans :: (log2(l) - log2(m) + 1).
 ex.
 k = 6,l = 8
 so initially m = 2;
 after 1st iteration m = 4;
 then break;
 so min = log2(8) - log2(4) + 1  = 3 -2 + 1 = 2.



 On Sat, Jul 2, 2011 at 1:16 AM, cegprakash cegprak...@gmail.com wrote:
  nope

  On Jul 2, 1:14 pm, keyan karthi keyankarthi1...@gmail.com wrote:
   yup :)

   On Sat, Jul 2, 2011 at 1:38 PM, Shalini Sah 
  shalinisah.luv4cod...@gmail.com

wrote:
i guess the no. of 1s in the binary representation of the number is the
answer..for 6 its 2...

On Sat, Jul 2, 2011 at 1:32 PM, cegprakash cegprak...@gmail.com
  wrote:

the length of the rope is l units.
I can only cut any rope into two halves.

for example if the length of the rope is 8 and we need a length of
rope 6

we first cut into two halves and we get 4, 4
now we cut any of the half again and we get 4,2,2

now we can merge 4 and 2 and form a rope of length 6.

in this example we need a minimum of 2 cuts to get the length of rope
6 from 8

assume that l is always a power of 2 and we need always a even length
of rope from it how to find the number of minimum cuts needed to get
the new rope?.

--
You received this message because you are subscribed to the Google
  Groups
Algorithm Geeks group.
To post to this group, send email to algogeeks@googlegroups.com.
To unsubscribe from this group, send email to
algogeeks+unsubscr...@googlegroups.com.
For more options, visit this group at
   http://groups.google.com/group/algogeeks?hl=en.

 --
You received this message because you are subscribed to the Google
  Groups
Algorithm Geeks group.
To post to this group, send email to algogeeks@googlegroups.com.
To unsubscribe from this group, send email to
algogeeks+unsubscr...@googlegroups.com.
For more options, visit this group at
   http://groups.google.com/group/algogeeks?hl=en.

  --
  You received this message because you are subscribed to the Google Groups
  Algorithm Geeks group.
  To post to this group, send email to algogeeks@googlegroups.com.
  To unsubscribe from this group, send email to
  algogeeks+unsubscr...@googlegroups.com.
  For more options, visit this group at
 http://groups.google.com/group/algogeeks?hl=en.

 --
 Varun Pahwa
 B.Tech (IT)
 7th Sem.
 Indian Institute of Information Technology Allahabad.
 Ph : 09793899112 ,08011820777
 Official Email :: rit2008...@iiita.ac.in
 Another Email :: varunpahwa.ii...@gmail.com

 People who fail to plan are those who plan to fail.

-- 
You received this message because you are subscribed to the Google Groups 
Algorithm Geeks group.
To post to this group, send email to algogeeks@googlegroups.com.
To unsubscribe from this group, send email to 
algogeeks+unsubscr...@googlegroups.com.
For more options, visit this group at 
http://groups.google.com/group/algogeeks?hl=en.



Re: [algogeeks] Re: help..

2011-07-02 Thread sunny agrawal
@varun
I think u want to write

while (k % m == 0)

On Sat, Jul 2, 2011 at 1:56 PM, varun pahwa varunpahwa2...@gmail.comwrote:

 k - rope of desired length.
 l - rope of given length
 m = 2;
 while(k % m)
 m *= 2;
 ans :: (log2(l) - log2(m) + 1).
 ex.
 k = 6,l = 8
 so initially m = 2;
 after 1st iteration m = 4;
 then break;
 so min = log2(8) - log2(4) + 1  = 3 -2 + 1 = 2.



 On Sat, Jul 2, 2011 at 1:16 AM, cegprakash cegprak...@gmail.com wrote:

 nope

 On Jul 2, 1:14 pm, keyan karthi keyankarthi1...@gmail.com wrote:
  yup :)
 
  On Sat, Jul 2, 2011 at 1:38 PM, Shalini Sah 
 shalinisah.luv4cod...@gmail.com
 
   wrote:
   i guess the no. of 1s in the binary representation of the number is
 the
   answer..for 6 its 2...
 
   On Sat, Jul 2, 2011 at 1:32 PM, cegprakash cegprak...@gmail.com
 wrote:
 
   the length of the rope is l units.
   I can only cut any rope into two halves.
 
   for example if the length of the rope is 8 and we need a length of
   rope 6
 
   we first cut into two halves and we get 4, 4
   now we cut any of the half again and we get 4,2,2
 
   now we can merge 4 and 2 and form a rope of length 6.
 
   in this example we need a minimum of 2 cuts to get the length of rope
   6 from 8
 
   assume that l is always a power of 2 and we need always a even length
   of rope from it how to find the number of minimum cuts needed to get
   the new rope?.
 
   --
   You received this message because you are subscribed to the Google
 Groups
   Algorithm Geeks group.
   To post to this group, send email to algogeeks@googlegroups.com.
   To unsubscribe from this group, send email to
   algogeeks+unsubscr...@googlegroups.com.
   For more options, visit this group at
  http://groups.google.com/group/algogeeks?hl=en.
 
--
   You received this message because you are subscribed to the Google
 Groups
   Algorithm Geeks group.
   To post to this group, send email to algogeeks@googlegroups.com.
   To unsubscribe from this group, send email to
   algogeeks+unsubscr...@googlegroups.com.
   For more options, visit this group at
  http://groups.google.com/group/algogeeks?hl=en.

 --
 You received this message because you are subscribed to the Google Groups
 Algorithm Geeks group.
 To post to this group, send email to algogeeks@googlegroups.com.
 To unsubscribe from this group, send email to
 algogeeks+unsubscr...@googlegroups.com.
 For more options, visit this group at
 http://groups.google.com/group/algogeeks?hl=en.




 --
 Varun Pahwa
 B.Tech (IT)
 7th Sem.
 Indian Institute of Information Technology Allahabad.
 Ph : 09793899112 ,08011820777
 Official Email :: rit2008...@iiita.ac.in
 Another Email :: varunpahwa.ii...@gmail.com

 People who fail to plan are those who plan to fail.

  --
 You received this message because you are subscribed to the Google Groups
 Algorithm Geeks group.
 To post to this group, send email to algogeeks@googlegroups.com.
 To unsubscribe from this group, send email to
 algogeeks+unsubscr...@googlegroups.com.
 For more options, visit this group at
 http://groups.google.com/group/algogeeks?hl=en.




-- 
Sunny Aggrawal
B-Tech IV year,CSI
Indian Institute Of Technology,Roorkee

-- 
You received this message because you are subscribed to the Google Groups 
Algorithm Geeks group.
To post to this group, send email to algogeeks@googlegroups.com.
To unsubscribe from this group, send email to 
algogeeks+unsubscr...@googlegroups.com.
For more options, visit this group at 
http://groups.google.com/group/algogeeks?hl=en.



[algogeeks] Re: help..

2011-07-02 Thread cegprakash
whats mean by first set bit and last set bit? do you simply mean the
index of first and last bit?

On Jul 2, 1:25 pm, sunny agrawal sunny816.i...@gmail.com wrote:
 xor the length of the rope with the required length and difference between
 the indexes of first set and last set bit *may* be the answer !!



 On Sat, Jul 2, 2011 at 1:46 PM, cegprakash cegprak...@gmail.com wrote:
  nope

  On Jul 2, 1:14 pm, keyan karthi keyankarthi1...@gmail.com wrote:
   yup :)

   On Sat, Jul 2, 2011 at 1:38 PM, Shalini Sah 
  shalinisah.luv4cod...@gmail.com

wrote:
i guess the no. of 1s in the binary representation of the number is the
answer..for 6 its 2...

On Sat, Jul 2, 2011 at 1:32 PM, cegprakash cegprak...@gmail.com
  wrote:

the length of the rope is l units.
I can only cut any rope into two halves.

for example if the length of the rope is 8 and we need a length of
rope 6

we first cut into two halves and we get 4, 4
now we cut any of the half again and we get 4,2,2

now we can merge 4 and 2 and form a rope of length 6.

in this example we need a minimum of 2 cuts to get the length of rope
6 from 8

assume that l is always a power of 2 and we need always a even length
of rope from it how to find the number of minimum cuts needed to get
the new rope?.

--
You received this message because you are subscribed to the Google
  Groups
Algorithm Geeks group.
To post to this group, send email to algogeeks@googlegroups.com.
To unsubscribe from this group, send email to
algogeeks+unsubscr...@googlegroups.com.
For more options, visit this group at
   http://groups.google.com/group/algogeeks?hl=en.

 --
You received this message because you are subscribed to the Google
  Groups
Algorithm Geeks group.
To post to this group, send email to algogeeks@googlegroups.com.
To unsubscribe from this group, send email to
algogeeks+unsubscr...@googlegroups.com.
For more options, visit this group at
   http://groups.google.com/group/algogeeks?hl=en.

  --
  You received this message because you are subscribed to the Google Groups
  Algorithm Geeks group.
  To post to this group, send email to algogeeks@googlegroups.com.
  To unsubscribe from this group, send email to
  algogeeks+unsubscr...@googlegroups.com.
  For more options, visit this group at
 http://groups.google.com/group/algogeeks?hl=en.

 --
 Sunny Aggrawal
 B-Tech IV year,CSI
 Indian Institute Of Technology,Roorkee

-- 
You received this message because you are subscribed to the Google Groups 
Algorithm Geeks group.
To post to this group, send email to algogeeks@googlegroups.com.
To unsubscribe from this group, send email to 
algogeeks+unsubscr...@googlegroups.com.
For more options, visit this group at 
http://groups.google.com/group/algogeeks?hl=en.



Re: [algogeeks] Re: help..

2011-07-02 Thread sunny agrawal
yes i have written that only
difference between indexes of first set bit and last set bit

On Sat, Jul 2, 2011 at 2:08 PM, cegprakash cegprak...@gmail.com wrote:

 whats mean by first set bit and last set bit? do you simply mean the
 index of first and last bit?

 On Jul 2, 1:25 pm, sunny agrawal sunny816.i...@gmail.com wrote:
  xor the length of the rope with the required length and difference
 between
  the indexes of first set and last set bit *may* be the answer !!
 
 
 
  On Sat, Jul 2, 2011 at 1:46 PM, cegprakash cegprak...@gmail.com wrote:
   nope
 
   On Jul 2, 1:14 pm, keyan karthi keyankarthi1...@gmail.com wrote:
yup :)
 
On Sat, Jul 2, 2011 at 1:38 PM, Shalini Sah 
   shalinisah.luv4cod...@gmail.com
 
 wrote:
 i guess the no. of 1s in the binary representation of the number is
 the
 answer..for 6 its 2...
 
 On Sat, Jul 2, 2011 at 1:32 PM, cegprakash cegprak...@gmail.com
   wrote:
 
 the length of the rope is l units.
 I can only cut any rope into two halves.
 
 for example if the length of the rope is 8 and we need a length of
 rope 6
 
 we first cut into two halves and we get 4, 4
 now we cut any of the half again and we get 4,2,2
 
 now we can merge 4 and 2 and form a rope of length 6.
 
 in this example we need a minimum of 2 cuts to get the length of
 rope
 6 from 8
 
 assume that l is always a power of 2 and we need always a even
 length
 of rope from it how to find the number of minimum cuts needed to
 get
 the new rope?.
 
 --
 You received this message because you are subscribed to the Google
   Groups
 Algorithm Geeks group.
 To post to this group, send email to algogeeks@googlegroups.com.
 To unsubscribe from this group, send email to
 algogeeks+unsubscr...@googlegroups.com.
 For more options, visit this group at
http://groups.google.com/group/algogeeks?hl=en.
 
  --
 You received this message because you are subscribed to the Google
   Groups
 Algorithm Geeks group.
 To post to this group, send email to algogeeks@googlegroups.com.
 To unsubscribe from this group, send email to
 algogeeks+unsubscr...@googlegroups.com.
 For more options, visit this group at
http://groups.google.com/group/algogeeks?hl=en.
 
   --
   You received this message because you are subscribed to the Google
 Groups
   Algorithm Geeks group.
   To post to this group, send email to algogeeks@googlegroups.com.
   To unsubscribe from this group, send email to
   algogeeks+unsubscr...@googlegroups.com.
   For more options, visit this group at
  http://groups.google.com/group/algogeeks?hl=en.
 
  --
  Sunny Aggrawal
  B-Tech IV year,CSI
  Indian Institute Of Technology,Roorkee

 --
 You received this message because you are subscribed to the Google Groups
 Algorithm Geeks group.
 To post to this group, send email to algogeeks@googlegroups.com.
 To unsubscribe from this group, send email to
 algogeeks+unsubscr...@googlegroups.com.
 For more options, visit this group at
 http://groups.google.com/group/algogeeks?hl=en.




-- 
Sunny Aggrawal
B-Tech IV year,CSI
Indian Institute Of Technology,Roorkee

-- 
You received this message because you are subscribed to the Google Groups 
Algorithm Geeks group.
To post to this group, send email to algogeeks@googlegroups.com.
To unsubscribe from this group, send email to 
algogeeks+unsubscr...@googlegroups.com.
For more options, visit this group at 
http://groups.google.com/group/algogeeks?hl=en.



Re: [algogeeks] Re: help..

2011-07-02 Thread sunny agrawal
l = 81 0 0 0
k = 6   0 1 1 0
xor  1 1 1 0
difference = 2

l = 161 0 0 0 0
k = 4 0 0 1 0 0
xor


On Sat, Jul 2, 2011 at 2:09 PM, sunny agrawal sunny816.i...@gmail.comwrote:

 yes i have written that only
 difference between indexes of first set bit and last set bit

 On Sat, Jul 2, 2011 at 2:08 PM, cegprakash cegprak...@gmail.com wrote:

 whats mean by first set bit and last set bit? do you simply mean the
 index of first and last bit?

 On Jul 2, 1:25 pm, sunny agrawal sunny816.i...@gmail.com wrote:
  xor the length of the rope with the required length and difference
 between
  the indexes of first set and last set bit *may* be the answer !!
 
 
 
  On Sat, Jul 2, 2011 at 1:46 PM, cegprakash cegprak...@gmail.com
 wrote:
   nope
 
   On Jul 2, 1:14 pm, keyan karthi keyankarthi1...@gmail.com wrote:
yup :)
 
On Sat, Jul 2, 2011 at 1:38 PM, Shalini Sah 
   shalinisah.luv4cod...@gmail.com
 
 wrote:
 i guess the no. of 1s in the binary representation of the number
 is the
 answer..for 6 its 2...
 
 On Sat, Jul 2, 2011 at 1:32 PM, cegprakash cegprak...@gmail.com
   wrote:
 
 the length of the rope is l units.
 I can only cut any rope into two halves.
 
 for example if the length of the rope is 8 and we need a length
 of
 rope 6
 
 we first cut into two halves and we get 4, 4
 now we cut any of the half again and we get 4,2,2
 
 now we can merge 4 and 2 and form a rope of length 6.
 
 in this example we need a minimum of 2 cuts to get the length of
 rope
 6 from 8
 
 assume that l is always a power of 2 and we need always a even
 length
 of rope from it how to find the number of minimum cuts needed to
 get
 the new rope?.
 
 --
 You received this message because you are subscribed to the
 Google
   Groups
 Algorithm Geeks group.
 To post to this group, send email to algogeeks@googlegroups.com.
 To unsubscribe from this group, send email to
 algogeeks+unsubscr...@googlegroups.com.
 For more options, visit this group at
http://groups.google.com/group/algogeeks?hl=en.
 
  --
 You received this message because you are subscribed to the Google
   Groups
 Algorithm Geeks group.
 To post to this group, send email to algogeeks@googlegroups.com.
 To unsubscribe from this group, send email to
 algogeeks+unsubscr...@googlegroups.com.
 For more options, visit this group at
http://groups.google.com/group/algogeeks?hl=en.
 
   --
   You received this message because you are subscribed to the Google
 Groups
   Algorithm Geeks group.
   To post to this group, send email to algogeeks@googlegroups.com.
   To unsubscribe from this group, send email to
   algogeeks+unsubscr...@googlegroups.com.
   For more options, visit this group at
  http://groups.google.com/group/algogeeks?hl=en.
 
  --
  Sunny Aggrawal
  B-Tech IV year,CSI
  Indian Institute Of Technology,Roorkee

 --
 You received this message because you are subscribed to the Google Groups
 Algorithm Geeks group.
 To post to this group, send email to algogeeks@googlegroups.com.
 To unsubscribe from this group, send email to
 algogeeks+unsubscr...@googlegroups.com.
 For more options, visit this group at
 http://groups.google.com/group/algogeeks?hl=en.




 --
 Sunny Aggrawal
 B-Tech IV year,CSI
 Indian Institute Of Technology,Roorkee




-- 
Sunny Aggrawal
B-Tech IV year,CSI
Indian Institute Of Technology,Roorkee

-- 
You received this message because you are subscribed to the Google Groups 
Algorithm Geeks group.
To post to this group, send email to algogeeks@googlegroups.com.
To unsubscribe from this group, send email to 
algogeeks+unsubscr...@googlegroups.com.
For more options, visit this group at 
http://groups.google.com/group/algogeeks?hl=en.



[algogeeks] Re: help..

2011-07-02 Thread cegprakash
even that won't work

for example:
if we need a length of rope 14 from a length of rope 16

according to varun's algo

initially m=2
14%2 is 0.. so m=4
14%4 is not 0.. break..

so log2(16)-log2(14)+ 1 == 4-3+1 = 2 which is wrong

but actually we need atleast 3 cuts.
16== 8 + 8 == 8+ 4+ 4 == 8 + 4+ 2 + 2.. now we can merge 2,4 and 8
to form 14



On Jul 2, 1:35 pm, sunny agrawal sunny816.i...@gmail.com wrote:
 @varun
 I think u want to write

 while (k % m == 0)

 On Sat, Jul 2, 2011 at 1:56 PM, varun pahwa varunpahwa2...@gmail.comwrote:



  k - rope of desired length.
  l - rope of given length
  m = 2;
  while(k % m)
  m *= 2;
  ans :: (log2(l) - log2(m) + 1).
  ex.
  k = 6,l = 8
  so initially m = 2;
  after 1st iteration m = 4;
  then break;
  so min = log2(8) - log2(4) + 1  = 3 -2 + 1 = 2.

  On Sat, Jul 2, 2011 at 1:16 AM, cegprakash cegprak...@gmail.com wrote:

  nope

  On Jul 2, 1:14 pm, keyan karthi keyankarthi1...@gmail.com wrote:
   yup :)

   On Sat, Jul 2, 2011 at 1:38 PM, Shalini Sah 
  shalinisah.luv4cod...@gmail.com

wrote:
i guess the no. of 1s in the binary representation of the number is
  the
answer..for 6 its 2...

On Sat, Jul 2, 2011 at 1:32 PM, cegprakash cegprak...@gmail.com
  wrote:

the length of the rope is l units.
I can only cut any rope into two halves.

for example if the length of the rope is 8 and we need a length of
rope 6

we first cut into two halves and we get 4, 4
now we cut any of the half again and we get 4,2,2

now we can merge 4 and 2 and form a rope of length 6.

in this example we need a minimum of 2 cuts to get the length of rope
6 from 8

assume that l is always a power of 2 and we need always a even length
of rope from it how to find the number of minimum cuts needed to get
the new rope?.

--
You received this message because you are subscribed to the Google
  Groups
Algorithm Geeks group.
To post to this group, send email to algogeeks@googlegroups.com.
To unsubscribe from this group, send email to
algogeeks+unsubscr...@googlegroups.com.
For more options, visit this group at
   http://groups.google.com/group/algogeeks?hl=en.

 --
You received this message because you are subscribed to the Google
  Groups
Algorithm Geeks group.
To post to this group, send email to algogeeks@googlegroups.com.
To unsubscribe from this group, send email to
algogeeks+unsubscr...@googlegroups.com.
For more options, visit this group at
   http://groups.google.com/group/algogeeks?hl=en.

  --
  You received this message because you are subscribed to the Google Groups
  Algorithm Geeks group.
  To post to this group, send email to algogeeks@googlegroups.com.
  To unsubscribe from this group, send email to
  algogeeks+unsubscr...@googlegroups.com.
  For more options, visit this group at
 http://groups.google.com/group/algogeeks?hl=en.

  --
  Varun Pahwa
  B.Tech (IT)
  7th Sem.
  Indian Institute of Information Technology Allahabad.
  Ph : 09793899112 ,08011820777
  Official Email :: rit2008...@iiita.ac.in
  Another Email :: varunpahwa.ii...@gmail.com

  People who fail to plan are those who plan to fail.

   --
  You received this message because you are subscribed to the Google Groups
  Algorithm Geeks group.
  To post to this group, send email to algogeeks@googlegroups.com.
  To unsubscribe from this group, send email to
  algogeeks+unsubscr...@googlegroups.com.
  For more options, visit this group at
 http://groups.google.com/group/algogeeks?hl=en.

 --
 Sunny Aggrawal
 B-Tech IV year,CSI
 Indian Institute Of Technology,Roorkee

-- 
You received this message because you are subscribed to the Google Groups 
Algorithm Geeks group.
To post to this group, send email to algogeeks@googlegroups.com.
To unsubscribe from this group, send email to 
algogeeks+unsubscr...@googlegroups.com.
For more options, visit this group at 
http://groups.google.com/group/algogeeks?hl=en.



[algogeeks] Re: help..

2011-07-02 Thread cegprakash
@varun: i think it works.. could u tell me how u found it

-- 
You received this message because you are subscribed to the Google Groups 
Algorithm Geeks group.
To post to this group, send email to algogeeks@googlegroups.com.
To unsubscribe from this group, send email to 
algogeeks+unsubscr...@googlegroups.com.
For more options, visit this group at 
http://groups.google.com/group/algogeeks?hl=en.



[algogeeks] Re: help..

2011-07-02 Thread cegprakash
@ sunny: so your's doesn't work right?

-- 
You received this message because you are subscribed to the Google Groups 
Algorithm Geeks group.
To post to this group, send email to algogeeks@googlegroups.com.
To unsubscribe from this group, send email to 
algogeeks+unsubscr...@googlegroups.com.
For more options, visit this group at 
http://groups.google.com/group/algogeeks?hl=en.



Re: [algogeeks] Re: help..

2011-07-02 Thread sunny agrawal
why ?

On Sat, Jul 2, 2011 at 2:20 PM, cegprakash cegprak...@gmail.com wrote:

 @ sunny: so your's doesn't work right?

 --
 You received this message because you are subscribed to the Google Groups
 Algorithm Geeks group.
 To post to this group, send email to algogeeks@googlegroups.com.
 To unsubscribe from this group, send email to
 algogeeks+unsubscr...@googlegroups.com.
 For more options, visit this group at
 http://groups.google.com/group/algogeeks?hl=en.




-- 
Sunny Aggrawal
B-Tech IV year,CSI
Indian Institute Of Technology,Roorkee

-- 
You received this message because you are subscribed to the Google Groups 
Algorithm Geeks group.
To post to this group, send email to algogeeks@googlegroups.com.
To unsubscribe from this group, send email to 
algogeeks+unsubscr...@googlegroups.com.
For more options, visit this group at 
http://groups.google.com/group/algogeeks?hl=en.



[algogeeks] Re: help..

2011-07-02 Thread cegprakash
@varun: explanation or proof for your soln. plz..

-- 
You received this message because you are subscribed to the Google Groups 
Algorithm Geeks group.
To post to this group, send email to algogeeks@googlegroups.com.
To unsubscribe from this group, send email to 
algogeeks+unsubscr...@googlegroups.com.
For more options, visit this group at 
http://groups.google.com/group/algogeeks?hl=en.



[algogeeks] Re: help..

2011-07-02 Thread cegprakash
oh fine.. got it now.. set bit is '1' right.. and is there any short
ways to find the difference between first set and short set bit
without dividing by 2 repeatedly?

-- 
You received this message because you are subscribed to the Google Groups 
Algorithm Geeks group.
To post to this group, send email to algogeeks@googlegroups.com.
To unsubscribe from this group, send email to 
algogeeks+unsubscr...@googlegroups.com.
For more options, visit this group at 
http://groups.google.com/group/algogeeks?hl=en.



Re: [algogeeks] Re: help..

2011-07-02 Thread sunny agrawal
for a number N
first set bit(From Left) is simply integer value of log(N)
last set bit can be calculated as

N = N-(N(N-1)); and then Log(N)

int i = log(n);
n -= n(n-1);
int j = log(n);

i-j will be the answer.


On Sat, Jul 2, 2011 at 2:34 PM, cegprakash cegprak...@gmail.com wrote:

 oh fine.. got it now.. set bit is '1' right.. and is there any short
 ways to find the difference between first set and short set bit
 without dividing by 2 repeatedly?

 --
 You received this message because you are subscribed to the Google Groups
 Algorithm Geeks group.
 To post to this group, send email to algogeeks@googlegroups.com.
 To unsubscribe from this group, send email to
 algogeeks+unsubscr...@googlegroups.com.
 For more options, visit this group at
 http://groups.google.com/group/algogeeks?hl=en.




-- 
Sunny Aggrawal
B-Tech IV year,CSI
Indian Institute Of Technology,Roorkee

-- 
You received this message because you are subscribed to the Google Groups 
Algorithm Geeks group.
To post to this group, send email to algogeeks@googlegroups.com.
To unsubscribe from this group, send email to 
algogeeks+unsubscr...@googlegroups.com.
For more options, visit this group at 
http://groups.google.com/group/algogeeks?hl=en.



[algogeeks] Re: help..

2011-07-02 Thread cegprakash
awesome!! thank you so much :)

On Jul 2, 2:11 pm, sunny agrawal sunny816.i...@gmail.com wrote:
 for a number N
 first set bit(From Left) is simply integer value of log(N)
 last set bit can be calculated as

 N = N-(N(N-1)); and then Log(N)

 int i = log(n);
 n -= n(n-1);
 int j = log(n);

 i-j will be the answer.

 On Sat, Jul 2, 2011 at 2:34 PM, cegprakash cegprak...@gmail.com wrote:
  oh fine.. got it now.. set bit is '1' right.. and is there any short
  ways to find the difference between first set and short set bit
  without dividing by 2 repeatedly?

  --
  You received this message because you are subscribed to the Google Groups
  Algorithm Geeks group.
  To post to this group, send email to algogeeks@googlegroups.com.
  To unsubscribe from this group, send email to
  algogeeks+unsubscr...@googlegroups.com.
  For more options, visit this group at
 http://groups.google.com/group/algogeeks?hl=en.

 --
 Sunny Aggrawal
 B-Tech IV year,CSI
 Indian Institute Of Technology,Roorkee

-- 
You received this message because you are subscribed to the Google Groups 
Algorithm Geeks group.
To post to this group, send email to algogeeks@googlegroups.com.
To unsubscribe from this group, send email to 
algogeeks+unsubscr...@googlegroups.com.
For more options, visit this group at 
http://groups.google.com/group/algogeeks?hl=en.



[algogeeks] Re: help..

2011-07-02 Thread cegprakash
btw what N = N-(N(N-1))  does actually

On Jul 2, 2:11 pm, sunny agrawal sunny816.i...@gmail.com wrote:
 for a number N
 first set bit(From Left) is simply integer value of log(N)
 last set bit can be calculated as

 N = N-(N(N-1)); and then Log(N)

 int i = log(n);
 n -= n(n-1);
 int j = log(n);

 i-j will be the answer.

 On Sat, Jul 2, 2011 at 2:34 PM, cegprakash cegprak...@gmail.com wrote:
  oh fine.. got it now.. set bit is '1' right.. and is there any short
  ways to find the difference between first set and short set bit
  without dividing by 2 repeatedly?

  --
  You received this message because you are subscribed to the Google Groups
  Algorithm Geeks group.
  To post to this group, send email to algogeeks@googlegroups.com.
  To unsubscribe from this group, send email to
  algogeeks+unsubscr...@googlegroups.com.
  For more options, visit this group at
 http://groups.google.com/group/algogeeks?hl=en.

 --
 Sunny Aggrawal
 B-Tech IV year,CSI
 Indian Institute Of Technology,Roorkee

-- 
You received this message because you are subscribed to the Google Groups 
Algorithm Geeks group.
To post to this group, send email to algogeeks@googlegroups.com.
To unsubscribe from this group, send email to 
algogeeks+unsubscr...@googlegroups.com.
For more options, visit this group at 
http://groups.google.com/group/algogeeks?hl=en.



Re: [algogeeks] Re: help..

2011-07-02 Thread sunny agrawal
try out with examples!!
u will surely get in 2-3 examples

N(N-1) is a very famous expression, used in counting set bits. see what
this expression return

On Sat, Jul 2, 2011 at 2:51 PM, cegprakash cegprak...@gmail.com wrote:

 btw what N = N-(N(N-1))  does actually

 On Jul 2, 2:11 pm, sunny agrawal sunny816.i...@gmail.com wrote:
  for a number N
  first set bit(From Left) is simply integer value of log(N)
  last set bit can be calculated as
 
  N = N-(N(N-1)); and then Log(N)
 
  int i = log(n);
  n -= n(n-1);
  int j = log(n);
 
  i-j will be the answer.
 
  On Sat, Jul 2, 2011 at 2:34 PM, cegprakash cegprak...@gmail.com wrote:
   oh fine.. got it now.. set bit is '1' right.. and is there any short
   ways to find the difference between first set and short set bit
   without dividing by 2 repeatedly?
 
   --
   You received this message because you are subscribed to the Google
 Groups
   Algorithm Geeks group.
   To post to this group, send email to algogeeks@googlegroups.com.
   To unsubscribe from this group, send email to
   algogeeks+unsubscr...@googlegroups.com.
   For more options, visit this group at
  http://groups.google.com/group/algogeeks?hl=en.
 
  --
  Sunny Aggrawal
  B-Tech IV year,CSI
  Indian Institute Of Technology,Roorkee

 --
 You received this message because you are subscribed to the Google Groups
 Algorithm Geeks group.
 To post to this group, send email to algogeeks@googlegroups.com.
 To unsubscribe from this group, send email to
 algogeeks+unsubscr...@googlegroups.com.
 For more options, visit this group at
 http://groups.google.com/group/algogeeks?hl=en.




-- 
Sunny Aggrawal
B-Tech IV year,CSI
Indian Institute Of Technology,Roorkee

-- 
You received this message because you are subscribed to the Google Groups 
Algorithm Geeks group.
To post to this group, send email to algogeeks@googlegroups.com.
To unsubscribe from this group, send email to 
algogeeks+unsubscr...@googlegroups.com.
For more options, visit this group at 
http://groups.google.com/group/algogeeks?hl=en.



[algogeeks] Re: Test Cases

2011-07-02 Thread Ritesh Srivastava
Asymptotic complexity can never be better than O(n).
But you can reduce the number of exact comparisons from  2n to 3n/2 .
Take pair of numbers in each iteration and compare them.
Then compare the smaller to Min and greater to Max .
This way, you have 3 comparisons for every iteration where the
number of iterations is n/2 so total number of comparison is 3n/2.

On Jul 2, 8:47 am, Hemanth Sagar hemanth2...@gmail.com wrote:
 @sameer
 Though the asymptotic time complexity is equal,i.e. O(n), the actual time
 complexity is also influenced by the constants. I claim that running two
 loops will be slower because there are now three operations that are to be
 done additionally than the single loop.

 Also, if you take the case of insertion sort (vs) merge sort, though the
 time complexity of merge sort (nlogn) is better than insertion sort(n^2),
 for small n values, insertion sort is better. This is because there is also
 a role played by the constants.

 Correct me if I am wrong !

 -
 Hemanth

 On Sat, Jul 2, 2011 at 8:20 AM, sameer.mut...@gmail.com 

 sameer.mut...@gmail.com wrote:
  @hemanth: doing it in seperate for loops or in a single for loop both
  results in O(n) time.It does not make a difference.

  I think there cannot be a solution better than O(n) as we need to traverse
  the array atleast once.

  On Fri, Jul 1, 2011 at 12:57 PM, Hemanth Sagar hemanth2...@gmail.comwrote:

  The most efficient code will be surely of O(n). This is because there is
  no way of finding
  the min and max without scanning the entire array atleast once.

  Is this so? or any counter arguments ?

  On Sat, Jul 2, 2011 at 1:06 AM, rajeevrvis 
  rajeev.open.1...@gmail.comwrote:

  Are there any Algorithms for finding efficient max differences ?

  On Jul 1, 11:42 pm, Hemanth 007 hemanth2...@gmail.com wrote:
   Hii,
   One improvement is that finding min and max can be done in a single
   run, rather than calling the  findMin and findMax separately.
   But yet the order is O(n);
   Is there any better soln than O(n);

   #includestdio.h
   #includelimits.h
   void findExtreme(int array[],int size,int* min,int* max){
           int i;
           for(i=0;isize;i++){
                   if(array[i]  *max)*max = array[i];
                   if(array[i]*min)*min=array[i];
           }

   }

   main(){
           int arr[]={0,609,211,432,31,};
           int max=INT_MIN,min=INT_MAX;
           findExtreme(arr,sizeof(arr)/sizeof(arr[0]),min,max);
           printf(%d %d ,max,min);
           printf(%d,max-min);}

   ~
   ~
   ~

  --
  You received this message because you are subscribed to the Google Groups
  Algorithm Geeks group.
  To post to this group, send email to algogeeks@googlegroups.com.
  To unsubscribe from this group, send email to
  algogeeks+unsubscr...@googlegroups.com.
  For more options, visit this group at
 http://groups.google.com/group/algogeeks?hl=en.

   --
  You received this message because you are subscribed to the Google Groups
  Algorithm Geeks group.
  To post to this group, send email to algogeeks@googlegroups.com.
  To unsubscribe from this group, send email to
  algogeeks+unsubscr...@googlegroups.com.
  For more options, visit this group at
 http://groups.google.com/group/algogeeks?hl=en.

   --
  You received this message because you are subscribed to the Google Groups
  Algorithm Geeks group.
  To post to this group, send email to algogeeks@googlegroups.com.
  To unsubscribe from this group, send email to
  algogeeks+unsubscr...@googlegroups.com.
  For more options, visit this group at
 http://groups.google.com/group/algogeeks?hl=en.

-- 
You received this message because you are subscribed to the Google Groups 
Algorithm Geeks group.
To post to this group, send email to algogeeks@googlegroups.com.
To unsubscribe from this group, send email to 
algogeeks+unsubscr...@googlegroups.com.
For more options, visit this group at 
http://groups.google.com/group/algogeeks?hl=en.



Re: [algogeeks] Re: help..

2011-07-02 Thread santosh mahto
@sunny

that will  work fine(xoring).
In place of Xoring u can also do OR of two number and find the distance
between fist set bit from left and first set bit from right,

Since bit operation is really fast operation so best algo this is of
complexity O(1);

Explanation How it works:

In l only one bit will be set. In m  multiple bit  would be set with all bit
before the bit set in l.
NOW suppose u divde l in 2 parts means u have shifted set bit of l to the
right.
dividing again will shift set bit in l to the right.
u have to divide till  set bit in l reached the position where first  bit of
m is set.

how many times u have shifted is count of  dividng the rope

its Simple.

u can check with any example


Thanks
Santosh
On Sat, Jul 2, 2011 at 2:41 PM, sunny agrawal sunny816.i...@gmail.comwrote:

 for a number N
 first set bit(From Left) is simply integer value of log(N)
 last set bit can be calculated as

 N = N-(N(N-1)); and then Log(N)

 int i = log(n);
 n -= n(n-1);
 int j = log(n);

 i-j will be the answer.



 On Sat, Jul 2, 2011 at 2:34 PM, cegprakash cegprak...@gmail.com wrote:

 oh fine.. got it now.. set bit is '1' right.. and is there any short
 ways to find the difference between first set and short set bit
 without dividing by 2 repeatedly?

 --
  You received this message because you are subscribed to the Google
 Groups Algorithm Geeks group.
 To post to this group, send email to algogeeks@googlegroups.com.
 To unsubscribe from this group, send email to
 algogeeks+unsubscr...@googlegroups.com.
 For more options, visit this group at
 http://groups.google.com/group/algogeeks?hl=en.




 --
 Sunny Aggrawal
 B-Tech IV year,CSI
 Indian Institute Of Technology,Roorkee

 --
  You received this message because you are subscribed to the Google Groups
 Algorithm Geeks group.
 To post to this group, send email to algogeeks@googlegroups.com.
 To unsubscribe from this group, send email to
 algogeeks+unsubscr...@googlegroups.com.
 For more options, visit this group at
 http://groups.google.com/group/algogeeks?hl=en.


-- 
You received this message because you are subscribed to the Google Groups 
Algorithm Geeks group.
To post to this group, send email to algogeeks@googlegroups.com.
To unsubscribe from this group, send email to 
algogeeks+unsubscr...@googlegroups.com.
For more options, visit this group at 
http://groups.google.com/group/algogeeks?hl=en.



Re: [algogeeks] Re: help..

2011-07-02 Thread santosh mahto
@sunny

the no of set bits in m will tell what all length(4,2 in above case)
are need to be merged.
e.g if if  m ==6 then m = 0110
 since bit set position are 2 and 1.
so length of rope  need to combine is 2^2=4 and 2^1 = 2;i.e 4 and 2

Thnaks
Santosh

On Sat, Jul 2, 2011 at 2:58 PM, santosh mahto santoshbit2...@gmail.comwrote:

 @sunny

 that will  work fine(xoring).
 In place of Xoring u can also do OR of two number and find the distance
 between fist set bit from left and first set bit from right,

 Since bit operation is really fast operation so best algo this is of
 complexity O(1);

 Explanation How it works:

 In l only one bit will be set. In m  multiple bit  would be set with all
 bit before the bit set in l.
 NOW suppose u divde l in 2 parts means u have shifted set bit of l to the
 right.
 dividing again will shift set bit in l to the right.
 u have to divide till  set bit in l reached the position where first  bit
 of m is set.

 how many times u have shifted is count of  dividng the rope

 its Simple.

 u can check with any example


 Thanks
 Santosh
   On Sat, Jul 2, 2011 at 2:41 PM, sunny agrawal 
 sunny816.i...@gmail.comwrote:

 for a number N
 first set bit(From Left) is simply integer value of log(N)
 last set bit can be calculated as

 N = N-(N(N-1)); and then Log(N)

 int i = log(n);
 n -= n(n-1);
 int j = log(n);

 i-j will be the answer.



 On Sat, Jul 2, 2011 at 2:34 PM, cegprakash cegprak...@gmail.com wrote:

 oh fine.. got it now.. set bit is '1' right.. and is there any short
 ways to find the difference between first set and short set bit
 without dividing by 2 repeatedly?

 --
  You received this message because you are subscribed to the Google
 Groups Algorithm Geeks group.
 To post to this group, send email to algogeeks@googlegroups.com.
 To unsubscribe from this group, send email to
 algogeeks+unsubscr...@googlegroups.com.
 For more options, visit this group at
 http://groups.google.com/group/algogeeks?hl=en.




 --
 Sunny Aggrawal
 B-Tech IV year,CSI
 Indian Institute Of Technology,Roorkee

 --
  You received this message because you are subscribed to the Google
 Groups Algorithm Geeks group.
 To post to this group, send email to algogeeks@googlegroups.com.
 To unsubscribe from this group, send email to
 algogeeks+unsubscr...@googlegroups.com.
 For more options, visit this group at
 http://groups.google.com/group/algogeeks?hl=en.




-- 
You received this message because you are subscribed to the Google Groups 
Algorithm Geeks group.
To post to this group, send email to algogeeks@googlegroups.com.
To unsubscribe from this group, send email to 
algogeeks+unsubscr...@googlegroups.com.
For more options, visit this group at 
http://groups.google.com/group/algogeeks?hl=en.



[algogeeks] Re: help..

2011-07-02 Thread cegprakash
@ sunny

21 is 0
32 is 2
43 is 0
5 4 is 4
65 is 4

I don't find anything

-- 
You received this message because you are subscribed to the Google Groups 
Algorithm Geeks group.
To post to this group, send email to algogeeks@googlegroups.com.
To unsubscribe from this group, send email to 
algogeeks+unsubscr...@googlegroups.com.
For more options, visit this group at 
http://groups.google.com/group/algogeeks?hl=en.



[algogeeks] Re: help..

2011-07-02 Thread cegprakash
power of 2 less than n right?

On Jul 2, 2:38 pm, cegprakash cegprak...@gmail.com wrote:
 @ sunny

 21 is 0
 32 is 2
 43 is 0
 5 4 is 4
 65 is 4

 I don't find anything

-- 
You received this message because you are subscribed to the Google Groups 
Algorithm Geeks group.
To post to this group, send email to algogeeks@googlegroups.com.
To unsubscribe from this group, send email to 
algogeeks+unsubscr...@googlegroups.com.
For more options, visit this group at 
http://groups.google.com/group/algogeeks?hl=en.



[algogeeks] Re: help..

2011-07-02 Thread cegprakash
no no.. it should be multiple of 2 less than n? even that doesn't
satisfies for 43

On Jul 2, 2:41 pm, cegprakash cegprak...@gmail.com wrote:
 power of 2 less than n right?

 On Jul 2, 2:38 pm, cegprakash cegprak...@gmail.com wrote:

  @ sunny

  21 is 0
  32 is 2
  43 is 0
  5 4 is 4
  65 is 4

  I don't find anything

-- 
You received this message because you are subscribed to the Google Groups 
Algorithm Geeks group.
To post to this group, send email to algogeeks@googlegroups.com.
To unsubscribe from this group, send email to 
algogeeks+unsubscr...@googlegroups.com.
For more options, visit this group at 
http://groups.google.com/group/algogeeks?hl=en.



Re: [algogeeks] Re: help..

2011-07-02 Thread mohit goel
May be this can work.give any counter example...
int count;
main()
{
  int l,rope,cuts;
  scanf(%d%d,l,rope);
  count =0;

   find_cuts(l,rope);
   printf(cuts needed is %d,count);
   getch();
   return 0;
   }

 int find_cuts(int l,int rope)

 {

if(l==rope)
return count;
 count++;
 printf(%d,count);
 l=l/2;
 if(l==rope)
 return count;
 if(ropel)
 rope =rope-l;

 find_cuts(l,rope);

 }

-- 
You received this message because you are subscribed to the Google Groups 
Algorithm Geeks group.
To post to this group, send email to algogeeks@googlegroups.com.
To unsubscribe from this group, send email to 
algogeeks+unsubscr...@googlegroups.com.
For more options, visit this group at 
http://groups.google.com/group/algogeeks?hl=en.



[algogeeks] Re: Sum to K problem

2011-07-02 Thread ross
@navneet: good one.. In my above explanation, u could keep track of
back pointers
so that u may want to print the subset containing the sum..

On Jul 2, 11:54 am, Navneet Gupta navneetn...@gmail.com wrote:
 Wrote the code for same.

 #includeiostream
 using namespace std;

 int max(int a,int b)
 {
         if(ab)
                 return a;
         else
                 return b;

 }

 int main()
 {
         int n,k;
         cout\nEnter the count of numbers :;
         cinn;

         //Set of Elements
         cout\nEnter the elements :;
         int *A;
         A = new int[n];
         for(int i = 0; in; i++)
                 cinA[i];

         //Input Sum Value
         cout\nEnter the value of k :;
         cink;

         //Matrix for holding boolean values for subproblems (max subproblems 
 upto nk)
         int **M;
         M = (int **)new int[n];
         for(int i=0; in; i++)
                 M[i] = new int[k+1];

         //Populate all the values to false
         for(int i=0; in; i++)
                 for(int j=0; j=k; j++)
                         M[i][j] = 0;

         for(int i=0; in; i++)
                 M[i][0] = true;

         for(int i=1; in; i++)
                 for(int j=0; j=k; j++)
                         if(j-A[i]=0)
                         {
                                 M[i][j] = max(M[i-1][j], M[i-1][j-A[i]]);
                         }

         if(M[n-1][k] == 1)
                 cout\nPossible Subset present;
         else
                 cout\nPossible Subset not present;

         cin.get();
         return 0;









 }
 On Fri, Jun 24, 2011 at 11:42 PM, ross jagadish1...@gmail.com wrote:
  This is the subset sum problem which is NP,.

  The DP is as follows,

  M[i,j] = 1 , if a subset of first i elements has a sum = j.
  else 0,
  The recurrence is,
  M[i,j] = max(M[i-1,j] , M[i-1,j-Ai]) where Ai is the ith element.

  You can maintain back pointers to keep track of previous elements so
  that
  the solution can be reconstructed from the DP.

  Once this matrix is populated till sum=k, Then, the column
  corresponding to sum=k, gives
  the answer. complexity o(nk).

  On Jun 20, 6:38 pm, Harshal hc4...@gmail.com wrote:
  The problem is NP. Complexity using DP will be O(nk), where n is number of
  elements and k is required sum.

  S[0]=true; //choose no element
  S[1...k] = false;
  for each number i in your input
   for s = k downto i
          if ( S[s - i] == true )
              S[s] = true;

  S[s] = true indicates a sum of i can be obtained from a subset of the set.
  To get the elements, we can make S[s] contain the list of numbers that
  contain the sum.
  On Mon, Jun 20, 2011 at 6:53 PM, Navneet Gupta 
  navneetn...@gmail.comwrote:

   Ideally, yes it can. Though i would be happy even if someone gives a
   good answer for non-negative values.

   On Mon, Jun 20, 2011 at 6:28 PM, Akshata Sharma
   akshatasharm...@gmail.com wrote:
@Navneet: does the set contains negative elements also?

On Mon, Jun 20, 2011 at 12:26 PM, pacific :-) pacific4...@gmail.com
   wrote:

@vaibhav : Please note that more than two numbers can sum upto k.

On Mon, Jun 20, 2011 at 12:21 PM, vaibhav shukla 
   vaibhav200...@gmail.com
wrote:

sort the array using merge sort : order nlogn
take the first element,subtract it with 'k' , then search the result
using binary search in rest of the array : order nlogn
hence u get two elements which sum up to K in order nlogn

On Mon, Jun 20, 2011 at 12:14 PM, Navneet Gupta 
navneetn...@gmail.com

wrote:

Right, in the worst case, complexity with be O(2^N).
So what are the possible optimizations here? Would building
   pre-computed
data structures with intermediate sum values help in finding such
   pairs in
less complexity? I think then we can apply dynamic programming to 
find
   such
pairs.

On Mon, Jun 20, 2011 at 12:09 PM, oppilas . 
   jatka.oppimi...@gmail.com
wrote:

I think its a NP problem. The solution complexity can go up O(2^N) 
in
worst case.

On Mon, Jun 20, 2011 at 11:55 AM, Navneet Gupta 
   navneetn...@gmail.com
wrote:

Given a set of integers , find a set of numbers such that they sum
   to
a given number k .
Notice the set of numbers can have 2 or more than 2 numbers.

--Navneet

--
You received this message because you are subscribed to the Google
Groups Algorithm Geeks group.
To post to this group, send email to algogeeks@googlegroups.com.
To unsubscribe from this group, send email to
algogeeks+unsubscr...@googlegroups.com.
For more options, visit this group at
   http://groups.google.com/group/algogeeks?hl=en.

--
You received this message because you are subscribed to the Google
Groups Algorithm Geeks group.
To post to this group, send email to algogeeks@googlegroups.com.
To unsubscribe from this group, send email to

Re: [algogeeks] Re: help..

2011-07-02 Thread sunny agrawal
@cegprakash
Expression resets the least significant set bit

On Sat, Jul 2, 2011 at 3:20 PM, mohit goel mohitgoel291...@gmail.comwrote:

 May be this can work.give any counter example...
 int count;
 main()
 {
   int l,rope,cuts;
   scanf(%d%d,l,rope);
   count =0;

find_cuts(l,rope);
printf(cuts needed is %d,count);
getch();
return 0;
}

  int find_cuts(int l,int rope)

  {

 if(l==rope)
 return count;
  count++;
  printf(%d,count);
  l=l/2;
  if(l==rope)
  return count;
  if(ropel)
  rope =rope-l;

  find_cuts(l,rope);


  }

 --
 You received this message because you are subscribed to the Google Groups
 Algorithm Geeks group.
 To post to this group, send email to algogeeks@googlegroups.com.
 To unsubscribe from this group, send email to
 algogeeks+unsubscr...@googlegroups.com.
 For more options, visit this group at
 http://groups.google.com/group/algogeeks?hl=en.




-- 
Sunny Aggrawal
B-Tech IV year,CSI
Indian Institute Of Technology,Roorkee

-- 
You received this message because you are subscribed to the Google Groups 
Algorithm Geeks group.
To post to this group, send email to algogeeks@googlegroups.com.
To unsubscribe from this group, send email to 
algogeeks+unsubscr...@googlegroups.com.
For more options, visit this group at 
http://groups.google.com/group/algogeeks?hl=en.



Re: [algogeeks] Re: help..

2011-07-02 Thread sameer.mut...@gmail.com
nn-1  is the expression to find out if n is a power of 2...If nn-1 returns
0 its a power of 2 else its not.
And what sunny said is also ryt

On Sat, Jul 2, 2011 at 3:47 PM, sunny agrawal sunny816.i...@gmail.comwrote:

 @cegprakash
 Expression resets the least significant set bit


 On Sat, Jul 2, 2011 at 3:20 PM, mohit goel mohitgoel291...@gmail.comwrote:

 May be this can work.give any counter example...
 int count;
 main()
 {
   int l,rope,cuts;
   scanf(%d%d,l,rope);
   count =0;

find_cuts(l,rope);
printf(cuts needed is %d,count);
getch();
return 0;
}

  int find_cuts(int l,int rope)

  {

 if(l==rope)
 return count;
  count++;
  printf(%d,count);
  l=l/2;
  if(l==rope)
  return count;
  if(ropel)
  rope =rope-l;

  find_cuts(l,rope);


  }

 --
 You received this message because you are subscribed to the Google Groups
 Algorithm Geeks group.
 To post to this group, send email to algogeeks@googlegroups.com.
 To unsubscribe from this group, send email to
 algogeeks+unsubscr...@googlegroups.com.
 For more options, visit this group at
 http://groups.google.com/group/algogeeks?hl=en.




 --
 Sunny Aggrawal
 B-Tech IV year,CSI
 Indian Institute Of Technology,Roorkee

  --
 You received this message because you are subscribed to the Google Groups
 Algorithm Geeks group.
 To post to this group, send email to algogeeks@googlegroups.com.
 To unsubscribe from this group, send email to
 algogeeks+unsubscr...@googlegroups.com.
 For more options, visit this group at
 http://groups.google.com/group/algogeeks?hl=en.


-- 
You received this message because you are subscribed to the Google Groups 
Algorithm Geeks group.
To post to this group, send email to algogeeks@googlegroups.com.
To unsubscribe from this group, send email to 
algogeeks+unsubscr...@googlegroups.com.
For more options, visit this group at 
http://groups.google.com/group/algogeeks?hl=en.



Re: [algogeeks] Sum to K problem

2011-07-02 Thread keyan karthi
http://en.wikipedia.org/wiki/Subset_sum_problem
http://en.wikipedia.org/wiki/Subset_sum_problem this should help u :)
knapsack like dp :)


On Mon, Jun 20, 2011 at 11:55 AM, Navneet Gupta navneetn...@gmail.comwrote:

 Given a set of integers , find a set of numbers such that they sum to a
 given number k .
 Notice the set of numbers can have 2 or more than 2 numbers.


 --Navneet

  --
 You received this message because you are subscribed to the Google Groups
 Algorithm Geeks group.
 To post to this group, send email to algogeeks@googlegroups.com.
 To unsubscribe from this group, send email to
 algogeeks+unsubscr...@googlegroups.com.
 For more options, visit this group at
 http://groups.google.com/group/algogeeks?hl=en.


-- 
You received this message because you are subscribed to the Google Groups 
Algorithm Geeks group.
To post to this group, send email to algogeeks@googlegroups.com.
To unsubscribe from this group, send email to 
algogeeks+unsubscr...@googlegroups.com.
For more options, visit this group at 
http://groups.google.com/group/algogeeks?hl=en.



[algogeeks] GATE 2011 C Ques

2011-07-02 Thread KK
10. What does the following fragment of C-program print?

char c[] = GATE2011;
char *p =c;
printf(%s, p + p[3] - p[1]);

 (A) GATE2011 (B) E2011 (C) 2011 (D) 011
Answer: - (C)

why is p[3] - p[1] returning 4 

-- 
You received this message because you are subscribed to the Google Groups 
Algorithm Geeks group.
To post to this group, send email to algogeeks@googlegroups.com.
To unsubscribe from this group, send email to 
algogeeks+unsubscr...@googlegroups.com.
For more options, visit this group at 
http://groups.google.com/group/algogeeks?hl=en.



Re: [algogeeks] GATE 2011 C Ques

2011-07-02 Thread abhijith reddy
p[3] = 'E'
p[1] = 'A'
p[3]-p[1] = 4
?

On Sat, Jul 2, 2011 at 7:10 PM, KK kunalkapadi...@gmail.com wrote:

 10. What does the following fragment of C-program print?

 char c[] = GATE2011;
 char *p =c;
 printf(%s, p + p[3] - p[1]);

  (A) GATE2011 (B) E2011 (C) 2011 (D) 011
 Answer: - (C)

 why is p[3] - p[1] returning 4 

 --
 You received this message because you are subscribed to the Google Groups
 Algorithm Geeks group.
 To post to this group, send email to algogeeks@googlegroups.com.
 To unsubscribe from this group, send email to
 algogeeks+unsubscr...@googlegroups.com.
 For more options, visit this group at
 http://groups.google.com/group/algogeeks?hl=en.



-- 
You received this message because you are subscribed to the Google Groups 
Algorithm Geeks group.
To post to this group, send email to algogeeks@googlegroups.com.
To unsubscribe from this group, send email to 
algogeeks+unsubscr...@googlegroups.com.
For more options, visit this group at 
http://groups.google.com/group/algogeeks?hl=en.



Re: [algogeeks] GATE 2011 C Ques

2011-07-02 Thread sameer.mut...@gmail.com
ASCII value of 'A' is 65 and Asciivalue of 'E' is 69.
69-65=4

On Sat, Jul 2, 2011 at 7:12 PM, abhijith reddy abhijith200...@gmail.comwrote:

 p[3] = 'E'
 p[1] = 'A'
 p[3]-p[1] = 4
 ?


 On Sat, Jul 2, 2011 at 7:10 PM, KK kunalkapadi...@gmail.com wrote:

 10. What does the following fragment of C-program print?

 char c[] = GATE2011;
 char *p =c;
 printf(%s, p + p[3] - p[1]);

  (A) GATE2011 (B) E2011 (C) 2011 (D) 011
 Answer: - (C)

 why is p[3] - p[1] returning 4 

 --
 You received this message because you are subscribed to the Google Groups
 Algorithm Geeks group.
 To post to this group, send email to algogeeks@googlegroups.com.
 To unsubscribe from this group, send email to
 algogeeks+unsubscr...@googlegroups.com.
 For more options, visit this group at
 http://groups.google.com/group/algogeeks?hl=en.


  --
 You received this message because you are subscribed to the Google Groups
 Algorithm Geeks group.
 To post to this group, send email to algogeeks@googlegroups.com.
 To unsubscribe from this group, send email to
 algogeeks+unsubscr...@googlegroups.com.
 For more options, visit this group at
 http://groups.google.com/group/algogeeks?hl=en.


-- 
You received this message because you are subscribed to the Google Groups 
Algorithm Geeks group.
To post to this group, send email to algogeeks@googlegroups.com.
To unsubscribe from this group, send email to 
algogeeks+unsubscr...@googlegroups.com.
For more options, visit this group at 
http://groups.google.com/group/algogeeks?hl=en.



[algogeeks] Re: GATE 2011 C Ques

2011-07-02 Thread KK
got it dont bother!!!
anyway thanx abhijith!!!

-- 
You received this message because you are subscribed to the Google Groups 
Algorithm Geeks group.
To post to this group, send email to algogeeks@googlegroups.com.
To unsubscribe from this group, send email to 
algogeeks+unsubscr...@googlegroups.com.
For more options, visit this group at 
http://groups.google.com/group/algogeeks?hl=en.



[algogeeks] Recurrence Relation

2011-07-02 Thread KK
3. Consider the following recurrence:

T(n) = 2T(n ^ (1/2)) + 1

Which one of the following is true?
(A) T(n) = (loglogn)
(B) T(n) = (logn)
(C) T(n) = (sqrt(n))
(D) T(n) = (n)

Can we solve this using master theorem???
any other way???

-- 
You received this message because you are subscribed to the Google Groups 
Algorithm Geeks group.
To post to this group, send email to algogeeks@googlegroups.com.
To unsubscribe from this group, send email to 
algogeeks+unsubscr...@googlegroups.com.
For more options, visit this group at 
http://groups.google.com/group/algogeeks?hl=en.



Re: [algogeeks] Recurrence Relation

2011-07-02 Thread aditya kumar
answer is option A .

On Sat, Jul 2, 2011 at 8:32 PM, KK kunalkapadi...@gmail.com wrote:

 3. Consider the following recurrence:

 T(n) = 2T(n ^ (1/2)) + 1

 Which one of the following is true?
 (A) T(n) = (loglogn)
 (B) T(n) = (logn)
 (C) T(n) = (sqrt(n))
 (D) T(n) = (n)

 Can we solve this using master theorem???
 any other way???

 --
 You received this message because you are subscribed to the Google Groups
 Algorithm Geeks group.
 To post to this group, send email to algogeeks@googlegroups.com.
 To unsubscribe from this group, send email to
 algogeeks+unsubscr...@googlegroups.com.
 For more options, visit this group at
 http://groups.google.com/group/algogeeks?hl=en.



-- 
You received this message because you are subscribed to the Google Groups 
Algorithm Geeks group.
To post to this group, send email to algogeeks@googlegroups.com.
To unsubscribe from this group, send email to 
algogeeks+unsubscr...@googlegroups.com.
For more options, visit this group at 
http://groups.google.com/group/algogeeks?hl=en.



Re: [algogeeks] Re: help..

2011-07-02 Thread varun pahwa
@sunny ya  i wanted to write the while(k % m == 0)

On Sat, Jul 2, 2011 at 3:47 AM, sameer.mut...@gmail.com 
sameer.mut...@gmail.com wrote:

 nn-1  is the expression to find out if n is a power of 2...If nn-1
 returns 0 its a power of 2 else its not.
 And what sunny said is also ryt


 On Sat, Jul 2, 2011 at 3:47 PM, sunny agrawal sunny816.i...@gmail.comwrote:

 @cegprakash
 Expression resets the least significant set bit


 On Sat, Jul 2, 2011 at 3:20 PM, mohit goel mohitgoel291...@gmail.comwrote:

 May be this can work.give any counter example...
 int count;
 main()
 {
   int l,rope,cuts;
   scanf(%d%d,l,rope);
   count =0;

find_cuts(l,rope);
printf(cuts needed is %d,count);
getch();
return 0;
}

  int find_cuts(int l,int rope)

  {

 if(l==rope)
 return count;
  count++;
  printf(%d,count);
  l=l/2;
  if(l==rope)
  return count;
  if(ropel)
  rope =rope-l;

  find_cuts(l,rope);


  }

 --
 You received this message because you are subscribed to the Google Groups
 Algorithm Geeks group.
 To post to this group, send email to algogeeks@googlegroups.com.
 To unsubscribe from this group, send email to
 algogeeks+unsubscr...@googlegroups.com.
 For more options, visit this group at
 http://groups.google.com/group/algogeeks?hl=en.




 --
 Sunny Aggrawal
 B-Tech IV year,CSI
 Indian Institute Of Technology,Roorkee

  --
 You received this message because you are subscribed to the Google Groups
 Algorithm Geeks group.
 To post to this group, send email to algogeeks@googlegroups.com.
 To unsubscribe from this group, send email to
 algogeeks+unsubscr...@googlegroups.com.
 For more options, visit this group at
 http://groups.google.com/group/algogeeks?hl=en.


  --
 You received this message because you are subscribed to the Google Groups
 Algorithm Geeks group.
 To post to this group, send email to algogeeks@googlegroups.com.
 To unsubscribe from this group, send email to
 algogeeks+unsubscr...@googlegroups.com.
 For more options, visit this group at
 http://groups.google.com/group/algogeeks?hl=en.




-- 
Varun Pahwa
B.Tech (IT)
7th Sem.
Indian Institute of Information Technology Allahabad.
Ph : 09793899112 ,08011820777
Official Email :: rit2008...@iiita.ac.in
Another Email :: varunpahwa.ii...@gmail.com

People who fail to plan are those who plan to fail.

-- 
You received this message because you are subscribed to the Google Groups 
Algorithm Geeks group.
To post to this group, send email to algogeeks@googlegroups.com.
To unsubscribe from this group, send email to 
algogeeks+unsubscr...@googlegroups.com.
For more options, visit this group at 
http://groups.google.com/group/algogeeks?hl=en.



Re: [algogeeks] Re: help..

2011-07-02 Thread varun pahwa
@sunny thnx for the correction.

On Sat, Jul 2, 2011 at 9:16 AM, varun pahwa varunpahwa2...@gmail.comwrote:

 @sunny ya  i wanted to write the while(k % m == 0)


 On Sat, Jul 2, 2011 at 3:47 AM, sameer.mut...@gmail.com 
 sameer.mut...@gmail.com wrote:

 nn-1  is the expression to find out if n is a power of 2...If nn-1
 returns 0 its a power of 2 else its not.
 And what sunny said is also ryt


 On Sat, Jul 2, 2011 at 3:47 PM, sunny agrawal sunny816.i...@gmail.comwrote:

 @cegprakash
 Expression resets the least significant set bit


  On Sat, Jul 2, 2011 at 3:20 PM, mohit goel 
 mohitgoel291...@gmail.comwrote:

 May be this can work.give any counter example...
 int count;
 main()
 {
   int l,rope,cuts;
   scanf(%d%d,l,rope);
   count =0;

find_cuts(l,rope);
printf(cuts needed is %d,count);
getch();
return 0;
}

  int find_cuts(int l,int rope)

  {

 if(l==rope)
 return count;
  count++;
  printf(%d,count);
  l=l/2;
  if(l==rope)
  return count;
  if(ropel)
  rope =rope-l;

  find_cuts(l,rope);


  }

 --
 You received this message because you are subscribed to the Google
 Groups Algorithm Geeks group.
 To post to this group, send email to algogeeks@googlegroups.com.
 To unsubscribe from this group, send email to
 algogeeks+unsubscr...@googlegroups.com.
 For more options, visit this group at
 http://groups.google.com/group/algogeeks?hl=en.




 --
 Sunny Aggrawal
 B-Tech IV year,CSI
 Indian Institute Of Technology,Roorkee

  --
 You received this message because you are subscribed to the Google Groups
 Algorithm Geeks group.
 To post to this group, send email to algogeeks@googlegroups.com.
 To unsubscribe from this group, send email to
 algogeeks+unsubscr...@googlegroups.com.
 For more options, visit this group at
 http://groups.google.com/group/algogeeks?hl=en.


  --
 You received this message because you are subscribed to the Google Groups
 Algorithm Geeks group.
 To post to this group, send email to algogeeks@googlegroups.com.
 To unsubscribe from this group, send email to
 algogeeks+unsubscr...@googlegroups.com.
 For more options, visit this group at
 http://groups.google.com/group/algogeeks?hl=en.




 --
 Varun Pahwa
 B.Tech (IT)
 7th Sem.
 Indian Institute of Information Technology Allahabad.
 Ph : 09793899112 ,08011820777
 Official Email :: rit2008...@iiita.ac.in
 Another Email :: varunpahwa.ii...@gmail.com

 People who fail to plan are those who plan to fail.




-- 
Varun Pahwa
B.Tech (IT)
7th Sem.
Indian Institute of Information Technology Allahabad.
Ph : 09793899112 ,08011820777
Official Email :: rit2008...@iiita.ac.in
Another Email :: varunpahwa.ii...@gmail.com

People who fail to plan are those who plan to fail.

-- 
You received this message because you are subscribed to the Google Groups 
Algorithm Geeks group.
To post to this group, send email to algogeeks@googlegroups.com.
To unsubscribe from this group, send email to 
algogeeks+unsubscr...@googlegroups.com.
For more options, visit this group at 
http://groups.google.com/group/algogeeks?hl=en.



[algogeeks] pointer increment problem can anyone tell why this output is coming?

2011-07-02 Thread Deoki Nandan
#includeiostream

using namespace std;

int main()
{
int intArray[]={1,2,3};
int *p=intArray;
cout*(p++);
return 0;
}

output :

1


-- 
**With Regards
Deoki Nandan Vishwakarma

*
*

-- 
You received this message because you are subscribed to the Google Groups 
Algorithm Geeks group.
To post to this group, send email to algogeeks@googlegroups.com.
To unsubscribe from this group, send email to 
algogeeks+unsubscr...@googlegroups.com.
For more options, visit this group at 
http://groups.google.com/group/algogeeks?hl=en.



Re: [algogeeks] pointer increment problem can anyone tell why this output is coming?

2011-07-02 Thread vaibhav shukla
ptr is pointing to array. *(ptr++) will give ptr and *ptr will give
1,further ptr will be incremented and will point to 2.

On Sat, Jul 2, 2011 at 10:17 PM, Deoki Nandan deok...@gmail.com wrote:

 #includeiostream

 using namespace std;

 int main()
 {
 int intArray[]={1,2,3};
 int *p=intArray;
 cout*(p++);
 return 0;
 }

 output :

 1


 --
 **With Regards
 Deoki Nandan Vishwakarma

 *
 *

  --
 You received this message because you are subscribed to the Google Groups
 Algorithm Geeks group.
 To post to this group, send email to algogeeks@googlegroups.com.
 To unsubscribe from this group, send email to
 algogeeks+unsubscr...@googlegroups.com.
 For more options, visit this group at
 http://groups.google.com/group/algogeeks?hl=en.




-- 
  best wishes!!
Vaibhav Shukla
DU-MCA

-- 
You received this message because you are subscribed to the Google Groups 
Algorithm Geeks group.
To post to this group, send email to algogeeks@googlegroups.com.
To unsubscribe from this group, send email to 
algogeeks+unsubscr...@googlegroups.com.
For more options, visit this group at 
http://groups.google.com/group/algogeeks?hl=en.



Re: [algogeeks] pointer increment problem can anyone tell why this output is coming?

2011-07-02 Thread Deoki Nandan
yes thanx a lot . becoz there is no sequence point and there is post
increment operator is used .


On Sat, Jul 2, 2011 at 10:22 PM, aditya kumar
aditya.kumar130...@gmail.comwrote:

 though you have put bracket over pointer but it will still be defrenced
 first and then the pointer will increemnet to point o next location . its
 post increement dats y.

 On Sat, Jul 2, 2011 at 10:17 PM, Deoki Nandan deok...@gmail.com wrote:

 #includeiostream

 using namespace std;

 int main()
 {
 int intArray[]={1,2,3};
 int *p=intArray;
 cout*(p++);
 return 0;
 }

 output :

 1


 --
 **With Regards
 Deoki Nandan Vishwakarma

 *
 *

  --
 You received this message because you are subscribed to the Google Groups
 Algorithm Geeks group.
 To post to this group, send email to algogeeks@googlegroups.com.
 To unsubscribe from this group, send email to
 algogeeks+unsubscr...@googlegroups.com.
 For more options, visit this group at
 http://groups.google.com/group/algogeeks?hl=en.


  --
 You received this message because you are subscribed to the Google Groups
 Algorithm Geeks group.
 To post to this group, send email to algogeeks@googlegroups.com.
 To unsubscribe from this group, send email to
 algogeeks+unsubscr...@googlegroups.com.
 For more options, visit this group at
 http://groups.google.com/group/algogeeks?hl=en.




-- 
**With Regards
Deoki Nandan Vishwakarma

*
*

-- 
You received this message because you are subscribed to the Google Groups 
Algorithm Geeks group.
To post to this group, send email to algogeeks@googlegroups.com.
To unsubscribe from this group, send email to 
algogeeks+unsubscr...@googlegroups.com.
For more options, visit this group at 
http://groups.google.com/group/algogeeks?hl=en.



[algogeeks] Re: Sum to K problem

2011-07-02 Thread mittal
Is there some way to find 3 elements whose sum equal to K in an array in 
linear time.
Sum of 2 numbers can be done in linear time using hash in O(n) time, what if 
for that case i am not allowed any extra space.

-- 
You received this message because you are subscribed to the Google Groups 
Algorithm Geeks group.
To view this discussion on the web visit 
https://groups.google.com/d/msg/algogeeks/-/90sRhX0Im5EJ.
To post to this group, send email to algogeeks@googlegroups.com.
To unsubscribe from this group, send email to 
algogeeks+unsubscr...@googlegroups.com.
For more options, visit this group at 
http://groups.google.com/group/algogeeks?hl=en.



[algogeeks] Re: 2 D array(dynamic allocation)

2011-07-02 Thread Sandeep Jain
Here's my solution.

int** allocateMatrix(int m, int n)
{
  int **rowList = (int**)malloc(sizeof(int)*m*n + sizeof(int*)*m);
  int *colList = (int*)(rowList+m);
  int i;
  for(i=0; im; i++)
  {
rowList[i] = colList+i*n;
  }
  return rowList;
}

And here's the main method to test/understand the allocation.

int main()
{
  int m=3, n=4;
  int **mat = allocateMatrix(m,n);
  int i, j, c=0;
  int wordCount= m*n+m; //sizeof(int)*4*5 + sizeof(int*)*4; counting
words so sizeof is not needed
  int* memMap = (int*)mat;

  // Fill array elements with some values to be able to test
  for(i=0; im; i++)
for(j=0; jn; j++)
 mat[i][j] = c++;

  printf(\nAddress\tValue\n);
  for(i=0; iwordCount; i++)
printf(\n%u\t== %u, memMap+i, memMap[i]);

  getchar();
}

-- 
You received this message because you are subscribed to the Google Groups 
Algorithm Geeks group.
To post to this group, send email to algogeeks@googlegroups.com.
To unsubscribe from this group, send email to 
algogeeks+unsubscr...@googlegroups.com.
For more options, visit this group at 
http://groups.google.com/group/algogeeks?hl=en.



Re: [algogeeks] Re: 2 D array(dynamic allocation)

2011-07-02 Thread vaibhav shukla
@sandeep sir: thnx... good 1 :)

On Sat, Jul 2, 2011 at 11:32 PM, Sandeep Jain sandeep6...@gmail.com wrote:

 Here's my solution.

 int** allocateMatrix(int m, int n)
 {
  int **rowList = (int**)malloc(sizeof(int)*m*n + sizeof(int*)*m);
  int *colList = (int*)(rowList+m);
  int i;
  for(i=0; im; i++)
  {
rowList[i] = colList+i*n;
  }
  return rowList;
 }

 And here's the main method to test/understand the allocation.

 int main()
 {
  int m=3, n=4;
  int **mat = allocateMatrix(m,n);
  int i, j, c=0;
  int wordCount= m*n+m; //sizeof(int)*4*5 + sizeof(int*)*4; counting
 words so sizeof is not needed
  int* memMap = (int*)mat;

  // Fill array elements with some values to be able to test
  for(i=0; im; i++)
for(j=0; jn; j++)
 mat[i][j] = c++;

  printf(\nAddress\tValue\n);
  for(i=0; iwordCount; i++)
printf(\n%u\t== %u, memMap+i, memMap[i]);

  getchar();
 }

 --
 You received this message because you are subscribed to the Google Groups
 Algorithm Geeks group.
 To post to this group, send email to algogeeks@googlegroups.com.
 To unsubscribe from this group, send email to
 algogeeks+unsubscr...@googlegroups.com.
 For more options, visit this group at
 http://groups.google.com/group/algogeeks?hl=en.




-- 
  best wishes!!
Vaibhav Shukla

-- 
You received this message because you are subscribed to the Google Groups 
Algorithm Geeks group.
To post to this group, send email to algogeeks@googlegroups.com.
To unsubscribe from this group, send email to 
algogeeks+unsubscr...@googlegroups.com.
For more options, visit this group at 
http://groups.google.com/group/algogeeks?hl=en.



[algogeeks] output plzzzz

2011-07-02 Thread HARSH PAHUJA
1)
#includestdio.h
int main()
{
extern int a;
a=20;
printf(%d,sizeof(a));
return 0;
}

2)
#includestdio.h
int main()
{
extern int a;
printf(%d,sizeof(a));
return 0;
}



-- 
HARSHIT PAHUJA
M.N.N.I.T.
ALLAHABAD

-- 
You received this message because you are subscribed to the Google Groups 
Algorithm Geeks group.
To post to this group, send email to algogeeks@googlegroups.com.
To unsubscribe from this group, send email to 
algogeeks+unsubscr...@googlegroups.com.
For more options, visit this group at 
http://groups.google.com/group/algogeeks?hl=en.



[algogeeks] Re: Recurrence Relation

2011-07-02 Thread KK
Answer (B)

  Let n = 2^m,  T(n) = T(2^m)
  Let T(2^m) =  S(m)
From the above two, T(n) = S(m)

S(m)  = 2S(m/2) + C1
Above expression is a binary tree traversal recursion whose time
complexity is (m). You can also prove using Master theorem.

S(m)  = (m)
 = (logn)  /* Since n = 2^m */
Now, let us go back to the original recursive function T(n)

  T(n)  = S(m)
= (Logn)

I din understand this its GATE 2006 Q...

-- 
You received this message because you are subscribed to the Google Groups 
Algorithm Geeks group.
To post to this group, send email to algogeeks@googlegroups.com.
To unsubscribe from this group, send email to 
algogeeks+unsubscr...@googlegroups.com.
For more options, visit this group at 
http://groups.google.com/group/algogeeks?hl=en.



Re: [algogeeks] output plzzzz

2011-07-02 Thread Piyush Sinha
i)  [Linker error] undefined reference to `a'

ii) 4

On 7/3/11, HARSH PAHUJA hpahuja.mn...@gmail.com wrote:
 1)
 #includestdio.h
 int main()
 {
 extern int a;
 a=20;
 printf(%d,sizeof(a));
 return 0;
 }

 2)
 #includestdio.h
 int main()
 {
 extern int a;
 printf(%d,sizeof(a));
 return 0;
 }



 --
 HARSHIT PAHUJA
 M.N.N.I.T.
 ALLAHABAD

 --
 You received this message because you are subscribed to the Google Groups
 Algorithm Geeks group.
 To post to this group, send email to algogeeks@googlegroups.com.
 To unsubscribe from this group, send email to
 algogeeks+unsubscr...@googlegroups.com.
 For more options, visit this group at
 http://groups.google.com/group/algogeeks?hl=en.




-- 
*Piyush Sinha*
*IIIT, Allahabad*
*+91-8792136657*
*+91-7483122727*
*https://www.facebook.com/profile.php?id=10655377926 *

-- 
You received this message because you are subscribed to the Google Groups 
Algorithm Geeks group.
To post to this group, send email to algogeeks@googlegroups.com.
To unsubscribe from this group, send email to 
algogeeks+unsubscr...@googlegroups.com.
For more options, visit this group at 
http://groups.google.com/group/algogeeks?hl=en.



Re: [algogeeks] output plzzzz

2011-07-02 Thread HARSH PAHUJA
@piyush  plz explain how are u getting this..

On Sat, Jul 2, 2011 at 12:10 PM, Piyush Sinha ecstasy.piy...@gmail.comwrote:

 i)  [Linker error] undefined reference to `a'

 ii) 4

 On 7/3/11, HARSH PAHUJA hpahuja.mn...@gmail.com wrote:
  1)
  #includestdio.h
  int main()
  {
  extern int a;
  a=20;
  printf(%d,sizeof(a));
  return 0;
  }
 
  2)
  #includestdio.h
  int main()
  {
  extern int a;
  printf(%d,sizeof(a));
  return 0;
  }
 
 
 
  --
  HARSHIT PAHUJA
  M.N.N.I.T.
  ALLAHABAD
 
  --
  You received this message because you are subscribed to the Google Groups
  Algorithm Geeks group.
  To post to this group, send email to algogeeks@googlegroups.com.
  To unsubscribe from this group, send email to
  algogeeks+unsubscr...@googlegroups.com.
  For more options, visit this group at
  http://groups.google.com/group/algogeeks?hl=en.
 
 


 --
 *Piyush Sinha*
 *IIIT, Allahabad*
 *+91-8792136657*
 *+91-7483122727*
 *https://www.facebook.com/profile.php?id=10655377926 *

 --
 You received this message because you are subscribed to the Google Groups
 Algorithm Geeks group.
 To post to this group, send email to algogeeks@googlegroups.com.
 To unsubscribe from this group, send email to
 algogeeks+unsubscr...@googlegroups.com.
 For more options, visit this group at
 http://groups.google.com/group/algogeeks?hl=en.




-- 
HARSHIT PAHUJA
M.N.N.I.T.
ALLAHABAD

-- 
You received this message because you are subscribed to the Google Groups 
Algorithm Geeks group.
To post to this group, send email to algogeeks@googlegroups.com.
To unsubscribe from this group, send email to 
algogeeks+unsubscr...@googlegroups.com.
For more options, visit this group at 
http://groups.google.com/group/algogeeks?hl=en.



[algogeeks] Taking Input

2011-07-02 Thread anonymous procrastination
Hello,

I came across a problem where the first line of the input specfies a
number n.
Then next n lines, every line contains a few(no limit) number
seperated by spaces.
Then I want to find the sm of numbers on each line.

eg.
INPUT
5
1 4 5
3 1 6
11 23
1 9 8 7 6
7 7 7

Now algorithm offcourse is not a problem. But I'm not getting how to
take input in C.
This is driving me crazy.
tried with scanf and getchar but unable to get it right.
Please help

-- 
You received this message because you are subscribed to the Google Groups 
Algorithm Geeks group.
To post to this group, send email to algogeeks@googlegroups.com.
To unsubscribe from this group, send email to 
algogeeks+unsubscr...@googlegroups.com.
For more options, visit this group at 
http://groups.google.com/group/algogeeks?hl=en.



Re: [algogeeks] Re: GATE 2011 C Ques

2011-07-02 Thread Wladimir Tavares
using pointer arithmetic we have to skip four characters then 2011 is
correct


Wladimir Araujo Tavares
*Federal University of Ceará

*




On Sat, Jul 2, 2011 at 11:00 AM, KK kunalkapadi...@gmail.com wrote:

 got it dont bother!!!
 anyway thanx abhijith!!!

 --
 You received this message because you are subscribed to the Google Groups
 Algorithm Geeks group.
 To post to this group, send email to algogeeks@googlegroups.com.
 To unsubscribe from this group, send email to
 algogeeks+unsubscr...@googlegroups.com.
 For more options, visit this group at
 http://groups.google.com/group/algogeeks?hl=en.



-- 
You received this message because you are subscribed to the Google Groups 
Algorithm Geeks group.
To post to this group, send email to algogeeks@googlegroups.com.
To unsubscribe from this group, send email to 
algogeeks+unsubscr...@googlegroups.com.
For more options, visit this group at 
http://groups.google.com/group/algogeeks?hl=en.



Re: [algogeeks] output plzzzz

2011-07-02 Thread HARSH PAHUJA
will ny1 xplain this extern concept
y this is a linker error?

On Sat, Jul 2, 2011 at 12:49 PM, HARSH PAHUJA hpahuja.mn...@gmail.comwrote:

 @piyush  plz explain how are u getting this..


 On Sat, Jul 2, 2011 at 12:10 PM, Piyush Sinha ecstasy.piy...@gmail.comwrote:

 i)  [Linker error] undefined reference to `a'

 ii) 4

 On 7/3/11, HARSH PAHUJA hpahuja.mn...@gmail.com wrote:
  1)
  #includestdio.h
  int main()
  {
  extern int a;
  a=20;
  printf(%d,sizeof(a));
  return 0;
  }
 
  2)
  #includestdio.h
  int main()
  {
  extern int a;
  printf(%d,sizeof(a));
  return 0;
  }
 
 
 
  --
  HARSHIT PAHUJA
  M.N.N.I.T.
  ALLAHABAD
 
  --
  You received this message because you are subscribed to the Google
 Groups
  Algorithm Geeks group.
  To post to this group, send email to algogeeks@googlegroups.com.
  To unsubscribe from this group, send email to
  algogeeks+unsubscr...@googlegroups.com.
  For more options, visit this group at
  http://groups.google.com/group/algogeeks?hl=en.
 
 


 --
 *Piyush Sinha*
 *IIIT, Allahabad*
 *+91-8792136657*
 *+91-7483122727*
 *https://www.facebook.com/profile.php?id=10655377926 *

 --
 You received this message because you are subscribed to the Google Groups
 Algorithm Geeks group.
 To post to this group, send email to algogeeks@googlegroups.com.
 To unsubscribe from this group, send email to
 algogeeks+unsubscr...@googlegroups.com.
 For more options, visit this group at
 http://groups.google.com/group/algogeeks?hl=en.




 --
 HARSHIT PAHUJA
 M.N.N.I.T.
 ALLAHABAD





-- 
HARSHIT PAHUJA
M.N.N.I.T.
ALLAHABAD

-- 
You received this message because you are subscribed to the Google Groups 
Algorithm Geeks group.
To post to this group, send email to algogeeks@googlegroups.com.
To unsubscribe from this group, send email to 
algogeeks+unsubscr...@googlegroups.com.
For more options, visit this group at 
http://groups.google.com/group/algogeeks?hl=en.



[algogeeks] Interview Question

2011-07-02 Thread mittal
Given two arrays of numbers, find if each of the two arrays have the same 
set of ntegers ? Suggest an algo which can run faster than NlogN without 
extra space?


-- 
You received this message because you are subscribed to the Google Groups 
Algorithm Geeks group.
To view this discussion on the web visit 
https://groups.google.com/d/msg/algogeeks/-/W921z1woFOQJ.
To post to this group, send email to algogeeks@googlegroups.com.
To unsubscribe from this group, send email to 
algogeeks+unsubscr...@googlegroups.com.
For more options, visit this group at 
http://groups.google.com/group/algogeeks?hl=en.



[algogeeks] Number of Comparisons!

2011-07-02 Thread Nitish Garg
How many comparisons are necessary to find the largest and smallest of a set 
of n distinct elements?
Let's discuss it for n = 10 and maybe we can generalize it from there.

-- 
You received this message because you are subscribed to the Google Groups 
Algorithm Geeks group.
To view this discussion on the web visit 
https://groups.google.com/d/msg/algogeeks/-/3HMDH51b-NcJ.
To post to this group, send email to algogeeks@googlegroups.com.
To unsubscribe from this group, send email to 
algogeeks+unsubscr...@googlegroups.com.
For more options, visit this group at 
http://groups.google.com/group/algogeeks?hl=en.



Re: [algogeeks] Taking Input

2011-07-02 Thread Piyush Kapoor
you can use gets() ,to read a line and then easily find the individual
numbers


On Sun, Jul 3, 2011 at 1:22 AM, anonymous procrastination 
opamp1...@gmail.com wrote:

 Hello,

 I came across a problem where the first line of the input specfies a
 number n.
 Then next n lines, every line contains a few(no limit) number
 seperated by spaces.
 Then I want to find the sm of numbers on each line.

 eg.
 INPUT
 5
 1 4 5
 3 1 6
 11 23
 1 9 8 7 6
 7 7 7

 Now algorithm offcourse is not a problem. But I'm not getting how to
 take input in C.
 This is driving me crazy.
 tried with scanf and getchar but unable to get it right.
 Please help

 --
 You received this message because you are subscribed to the Google Groups
 Algorithm Geeks group.
 To post to this group, send email to algogeeks@googlegroups.com.
 To unsubscribe from this group, send email to
 algogeeks+unsubscr...@googlegroups.com.
 For more options, visit this group at
 http://groups.google.com/group/algogeeks?hl=en.




-- 
*Regards,*
*Piyush Kapoor,*
*CSE-IT-BHU*

-- 
You received this message because you are subscribed to the Google Groups 
Algorithm Geeks group.
To post to this group, send email to algogeeks@googlegroups.com.
To unsubscribe from this group, send email to 
algogeeks+unsubscr...@googlegroups.com.
For more options, visit this group at 
http://groups.google.com/group/algogeeks?hl=en.



[algogeeks] Re: 2 D array(dynamic allocation)

2011-07-02 Thread Gene
Unfortunately this invokes undefined behavior, so it may not work for
all architectures and compilers.  It relies on pointers to int having
the same alignment as int.

By far the best way to do this is use C99 features:

double (*a)[n_cols] = calloc(n_rows, sizeof *a);

If you don't have C99 and the row length is fixed, then you can
allocated a 2d array very easily:

#include stdio.h
#include stdlib.h

#define ROW_SIZE 10
typedef double ROW[ROW_SIZE];

int main(void)
{
  int i, j n_rows = 15;
  ROW *array = malloc(n_rows * sizeof(ROW));
  for (i  = 0; i  n_rows; i++)
for (j = 0; j  ROW_SIZE; j++)
  array[i][j] = 100 * i + j;
  for (i  = 0; i  n_rows; i++) {
for (j = 0; j  ROW_SIZE; j++)
  printf(%5.0f, array[i][j]);
printf(\n);
  }
  return 0;
}

If you need both array axes to be variable, then the norm is to
allocate a 1d array and define a macro to allow 2d access:

#include stdio.h
#include stdlib.h

typedef double *ARRAY;
#define Elt(A, I, J, NCOLS)  ((A)[(I) * (NCOLS) + (J)])

int no_fixed(void)
{
  int i, j, n_rows = 15, n_cols = 10;
  ARRAY array = malloc(n_rows * n_cols * sizeof array[0]);
  for (i  = 0; i  n_rows; i++)
for (j = 0; j  n_cols; j++)
  Elt(array, i, j, n_cols) = 100 * i + j;
  for (i  = 0; i  n_rows; i++) {
for (j = 0; j  n_cols; j++)
  printf(%5.0f, Elt(array, i, j, n_cols));
printf(\n);
  }
  return 0;
}


On Jul 2, 2:05 pm, vaibhav shukla vaibhav200...@gmail.com wrote:
 @sandeep sir: thnx... good 1 :)





 On Sat, Jul 2, 2011 at 11:32 PM, Sandeep Jain sandeep6...@gmail.com wrote:
  Here's my solution.

  int** allocateMatrix(int m, int n)
  {
   int **rowList = (int**)malloc(sizeof(int)*m*n + sizeof(int*)*m);
   int *colList = (int*)(rowList+m);
   int i;
   for(i=0; im; i++)
   {
     rowList[i] = colList+i*n;
   }
   return rowList;
  }

  And here's the main method to test/understand the allocation.

  int main()
  {
   int m=3, n=4;
   int **mat = allocateMatrix(m,n);
   int i, j, c=0;
   int wordCount= m*n+m; //sizeof(int)*4*5 + sizeof(int*)*4; counting
  words so sizeof is not needed
   int* memMap = (int*)mat;

   // Fill array elements with some values to be able to test
   for(i=0; im; i++)
     for(j=0; jn; j++)
      mat[i][j] = c++;

   printf(\nAddress\t    Value\n);
   for(i=0; iwordCount; i++)
     printf(\n%u\t== %u, memMap+i, memMap[i]);

   getchar();
  }

  --
  You received this message because you are subscribed to the Google Groups
  Algorithm Geeks group.
  To post to this group, send email to algogeeks@googlegroups.com.
  To unsubscribe from this group, send email to
  algogeeks+unsubscr...@googlegroups.com.
  For more options, visit this group at
 http://groups.google.com/group/algogeeks?hl=en.

 --
   best wishes!!
 Vaibhav Shukla- Hide quoted text -

 - Show quoted text -

-- 
You received this message because you are subscribed to the Google Groups 
Algorithm Geeks group.
To post to this group, send email to algogeeks@googlegroups.com.
To unsubscribe from this group, send email to 
algogeeks+unsubscr...@googlegroups.com.
For more options, visit this group at 
http://groups.google.com/group/algogeeks?hl=en.



Re: [algogeeks] Number of Comparisons!

2011-07-02 Thread udit sharma
If n is odd, then we perform
3*n/2 comparisons. If n is even, we perform 1 + 3*(n-2)/2



-- 
Regards
 UDIT
 DU- MCA

-- 
You received this message because you are subscribed to the Google Groups 
Algorithm Geeks group.
To post to this group, send email to algogeeks@googlegroups.com.
To unsubscribe from this group, send email to 
algogeeks+unsubscr...@googlegroups.com.
For more options, visit this group at 
http://groups.google.com/group/algogeeks?hl=en.



Re: [algogeeks] output plzzzz

2011-07-02 Thread santosh mahto
this will help

-sizeof operator only need declaration not definition.since sizeof operator
calculates size at
compile time.
At compile time code doesnot seek for definition.it only need declaration.if
any variable is declared with extern,it thinks that
its definition will be found at linking time.

In ur question:
1 case. when a =20 is done compiler see that its value is declared.so its OK
.but at link time since def of a is not found
so linker error. since linker tries to find the actual memory address where
the def of a is.

see after compilation the symbol table for a will be as

symbolname| memory location| value
 a |find later(garbage)| 20

so there is no problem with compiler .but at link time  garbage value need
to be filled with actual memory address
since def is not there so linker error.

case2. there will be noentry in the symbol table for a. as code doesnot
include any defintion related operation.


iF anything helps you, dont forget to say thanks

Thanks
Santosh





On Sun, Jul 3, 2011 at 1:38 AM, HARSH PAHUJA hpahuja.mn...@gmail.comwrote:

 will ny1 xplain this extern concept
 y this is a linker error?


 On Sat, Jul 2, 2011 at 12:49 PM, HARSH PAHUJA hpahuja.mn...@gmail.comwrote:

 @piyush  plz explain how are u getting this..


 On Sat, Jul 2, 2011 at 12:10 PM, Piyush Sinha 
 ecstasy.piy...@gmail.comwrote:

 i)  [Linker error] undefined reference to `a'

 ii) 4

 On 7/3/11, HARSH PAHUJA hpahuja.mn...@gmail.com wrote:
  1)
  #includestdio.h
  int main()
  {
  extern int a;
  a=20;
  printf(%d,sizeof(a));
  return 0;
  }
 
  2)
  #includestdio.h
  int main()
  {
  extern int a;
  printf(%d,sizeof(a));
  return 0;
  }
 
 
 
  --
  HARSHIT PAHUJA
  M.N.N.I.T.
  ALLAHABAD
 
  --
  You received this message because you are subscribed to the Google
 Groups
  Algorithm Geeks group.
  To post to this group, send email to algogeeks@googlegroups.com.
  To unsubscribe from this group, send email to
  algogeeks+unsubscr...@googlegroups.com.
  For more options, visit this group at
  http://groups.google.com/group/algogeeks?hl=en.
 
 


 --
 *Piyush Sinha*
 *IIIT, Allahabad*
 *+91-8792136657*
 *+91-7483122727*
 *https://www.facebook.com/profile.php?id=10655377926 *

 --
 You received this message because you are subscribed to the Google Groups
 Algorithm Geeks group.
 To post to this group, send email to algogeeks@googlegroups.com.
 To unsubscribe from this group, send email to
 algogeeks+unsubscr...@googlegroups.com.
 For more options, visit this group at
 http://groups.google.com/group/algogeeks?hl=en.




 --
  HARSHIT PAHUJA
 M.N.N.I.T.
 ALLAHABAD





 --
 HARSHIT PAHUJA
 M.N.N.I.T.
 ALLAHABAD


   --
 You received this message because you are subscribed to the Google Groups
 Algorithm Geeks group.
 To post to this group, send email to algogeeks@googlegroups.com.
 To unsubscribe from this group, send email to
 algogeeks+unsubscr...@googlegroups.com.
 For more options, visit this group at
 http://groups.google.com/group/algogeeks?hl=en.


-- 
You received this message because you are subscribed to the Google Groups 
Algorithm Geeks group.
To post to this group, send email to algogeeks@googlegroups.com.
To unsubscribe from this group, send email to 
algogeeks+unsubscr...@googlegroups.com.
For more options, visit this group at 
http://groups.google.com/group/algogeeks?hl=en.



[algogeeks] Re: Interview Question

2011-07-02 Thread Dumanshu
xor all the elements of both arrays ==0
sum of 1st array == sum of 2nd array
no. of elements in 1st == no. of elements in 2nd
if the above conditions are met, they have the same set.
m i missin sth?
On Jul 3, 1:23 am, mittal mohitm.1...@gmail.com wrote:
 Given two arrays of numbers, find if each of the two arrays have the same
 set of ntegers ? Suggest an algo which can run faster than NlogN without
 extra space?

-- 
You received this message because you are subscribed to the Google Groups 
Algorithm Geeks group.
To post to this group, send email to algogeeks@googlegroups.com.
To unsubscribe from this group, send email to 
algogeeks+unsubscr...@googlegroups.com.
For more options, visit this group at 
http://groups.google.com/group/algogeeks?hl=en.



Re: [algogeeks] Re: Interview Question

2011-07-02 Thread aditya kumar
xor will only result if corresponding elements are same . what if in both
the array set of integers are same but they arnt corresponding to each other
??

On Sun, Jul 3, 2011 at 2:37 AM, Dumanshu duman...@gmail.com wrote:

 xor all the elements of both arrays ==0
 sum of 1st array == sum of 2nd array
 no. of elements in 1st == no. of elements in 2nd
 if the above conditions are met, they have the same set.
 m i missin sth?
 On Jul 3, 1:23 am, mittal mohitm.1...@gmail.com wrote:
  Given two arrays of numbers, find if each of the two arrays have the same
  set of ntegers ? Suggest an algo which can run faster than NlogN without
  extra space?

 --
 You received this message because you are subscribed to the Google Groups
 Algorithm Geeks group.
 To post to this group, send email to algogeeks@googlegroups.com.
 To unsubscribe from this group, send email to
 algogeeks+unsubscr...@googlegroups.com.
 For more options, visit this group at
 http://groups.google.com/group/algogeeks?hl=en.



-- 
You received this message because you are subscribed to the Google Groups 
Algorithm Geeks group.
To post to this group, send email to algogeeks@googlegroups.com.
To unsubscribe from this group, send email to 
algogeeks+unsubscr...@googlegroups.com.
For more options, visit this group at 
http://groups.google.com/group/algogeeks?hl=en.



[algogeeks] Re: Taking Input

2011-07-02 Thread anonymous procrastination
#includestdio.h
#includestdlib.h
#define INF 9
int main()
{
int no,i,j=0,num=0;
int adj[102][102]={INF};
char neighbours[103],c;
scanf(%d,no);
for(i=1;i=no;i++)
{
   j=0;
gets(neighbours);
puts(neighbours);

}

system(PAUSE);
}


Can you please tell me why the following code is taking only (no-1)
strings as input?

-- 
You received this message because you are subscribed to the Google Groups 
Algorithm Geeks group.
To post to this group, send email to algogeeks@googlegroups.com.
To unsubscribe from this group, send email to 
algogeeks+unsubscr...@googlegroups.com.
For more options, visit this group at 
http://groups.google.com/group/algogeeks?hl=en.



Re: [algogeeks] Re: Interview Question

2011-07-02 Thread mohit mittal
Dont think that the corresponding elements should be same.
XOR Should do it anyway.

Btw other question How would you find the second largest element in an
array using minimum no of comparisons?Any thing better than O(n).?


On Sun, Jul 3, 2011 at 2:41 AM, aditya kumar
aditya.kumar130...@gmail.comwrote:

 xor will only result if corresponding elements are same . what if in both
 the array set of integers are same but they arnt corresponding to each other
 ??


 On Sun, Jul 3, 2011 at 2:37 AM, Dumanshu duman...@gmail.com wrote:

 xor all the elements of both arrays ==0
 sum of 1st array == sum of 2nd array
 no. of elements in 1st == no. of elements in 2nd
 if the above conditions are met, they have the same set.
 m i missin sth?
 On Jul 3, 1:23 am, mittal mohitm.1...@gmail.com wrote:
  Given two arrays of numbers, find if each of the two arrays have the
 same
  set of ntegers ? Suggest an algo which can run faster than NlogN without
  extra space?

 --
 You received this message because you are subscribed to the Google Groups
 Algorithm Geeks group.
 To post to this group, send email to algogeeks@googlegroups.com.
 To unsubscribe from this group, send email to
 algogeeks+unsubscr...@googlegroups.com.
 For more options, visit this group at
 http://groups.google.com/group/algogeeks?hl=en.


  --
 You received this message because you are subscribed to the Google Groups
 Algorithm Geeks group.
 To post to this group, send email to algogeeks@googlegroups.com.
 To unsubscribe from this group, send email to
 algogeeks+unsubscr...@googlegroups.com.
 For more options, visit this group at
 http://groups.google.com/group/algogeeks?hl=en.




-- 
Mohit Mittal
4th year , Computer Engineering
Student-Coordinator , DTU WebTeam
Delhi Technological University

-- 
You received this message because you are subscribed to the Google Groups 
Algorithm Geeks group.
To post to this group, send email to algogeeks@googlegroups.com.
To unsubscribe from this group, send email to 
algogeeks+unsubscr...@googlegroups.com.
For more options, visit this group at 
http://groups.google.com/group/algogeeks?hl=en.



[algogeeks] Re: Taking Input

2011-07-02 Thread anonymous procrastination
Hey Thanks,,
I got it.. when I press enter after entering the number it takes that
as an empty string.

-- 
You received this message because you are subscribed to the Google Groups 
Algorithm Geeks group.
To post to this group, send email to algogeeks@googlegroups.com.
To unsubscribe from this group, send email to 
algogeeks+unsubscr...@googlegroups.com.
For more options, visit this group at 
http://groups.google.com/group/algogeeks?hl=en.



Re: [algogeeks] Re: Interview Question

2011-07-02 Thread aditya kumar
@mohit..:i dint get the logic behind XOR plz explain ..nd ya i dont think
dat you can find second largest in less than O(n).

On Sun, Jul 3, 2011 at 2:43 AM, mohit mittal mohitm.1...@gmail.com wrote:

 Dont think that the corresponding elements should be same.
 XOR Should do it anyway.

 Btw other question How would you find the second largest element in an
 array using minimum no of comparisons?Any thing better than O(n).?


 On Sun, Jul 3, 2011 at 2:41 AM, aditya kumar aditya.kumar130...@gmail.com
  wrote:

 xor will only result if corresponding elements are same . what if in both
 the array set of integers are same but they arnt corresponding to each other
 ??


 On Sun, Jul 3, 2011 at 2:37 AM, Dumanshu duman...@gmail.com wrote:

 xor all the elements of both arrays ==0
 sum of 1st array == sum of 2nd array
 no. of elements in 1st == no. of elements in 2nd
 if the above conditions are met, they have the same set.
 m i missin sth?
 On Jul 3, 1:23 am, mittal mohitm.1...@gmail.com wrote:
  Given two arrays of numbers, find if each of the two arrays have the
 same
  set of ntegers ? Suggest an algo which can run faster than NlogN
 without
  extra space?

 --
 You received this message because you are subscribed to the Google Groups
 Algorithm Geeks group.
 To post to this group, send email to algogeeks@googlegroups.com.
 To unsubscribe from this group, send email to
 algogeeks+unsubscr...@googlegroups.com.
 For more options, visit this group at
 http://groups.google.com/group/algogeeks?hl=en.


  --
 You received this message because you are subscribed to the Google Groups
 Algorithm Geeks group.
 To post to this group, send email to algogeeks@googlegroups.com.
 To unsubscribe from this group, send email to
 algogeeks+unsubscr...@googlegroups.com.
 For more options, visit this group at
 http://groups.google.com/group/algogeeks?hl=en.




 --
 Mohit Mittal
 4th year , Computer Engineering
 Student-Coordinator , DTU WebTeam
 Delhi Technological University

  --
 You received this message because you are subscribed to the Google Groups
 Algorithm Geeks group.
 To post to this group, send email to algogeeks@googlegroups.com.
 To unsubscribe from this group, send email to
 algogeeks+unsubscr...@googlegroups.com.
 For more options, visit this group at
 http://groups.google.com/group/algogeeks?hl=en.


-- 
You received this message because you are subscribed to the Google Groups 
Algorithm Geeks group.
To post to this group, send email to algogeeks@googlegroups.com.
To unsubscribe from this group, send email to 
algogeeks+unsubscr...@googlegroups.com.
For more options, visit this group at 
http://groups.google.com/group/algogeeks?hl=en.



Re: [algogeeks] output plzzzz

2011-07-02 Thread HARSH PAHUJA
thanx i got it :)

On Sat, Jul 2, 2011 at 1:57 PM, santosh mahto santoshbit2...@gmail.comwrote:

 this will help

 -sizeof operator only need declaration not definition.since sizeof
 operator calculates size at
 compile time.
 At compile time code doesnot seek for definition.it only need
 declaration.if any variable is declared with extern,it thinks that
 its definition will be found at linking time.

 In ur question:
 1 case. when a =20 is done compiler see that its value is declared.so its
 OK .but at link time since def of a is not found
 so linker error. since linker tries to find the actual memory address where
 the def of a is.

 see after compilation the symbol table for a will be as

 symbolname| memory location| value
  a |find later(garbage)| 20

 so there is no problem with compiler .but at link time  garbage value need
 to be filled with actual memory address
 since def is not there so linker error.

 case2. there will be noentry in the symbol table for a. as code doesnot
 include any defintion related operation.


 iF anything helps you, dont forget to say thanks

 Thanks
 Santosh





 On Sun, Jul 3, 2011 at 1:38 AM, HARSH PAHUJA hpahuja.mn...@gmail.comwrote:

 will ny1 xplain this extern concept
 y this is a linker error?


 On Sat, Jul 2, 2011 at 12:49 PM, HARSH PAHUJA hpahuja.mn...@gmail.comwrote:

 @piyush  plz explain how are u getting this..


 On Sat, Jul 2, 2011 at 12:10 PM, Piyush Sinha 
 ecstasy.piy...@gmail.comwrote:

 i)  [Linker error] undefined reference to `a'

 ii) 4

 On 7/3/11, HARSH PAHUJA hpahuja.mn...@gmail.com wrote:
  1)
  #includestdio.h
  int main()
  {
  extern int a;
  a=20;
  printf(%d,sizeof(a));
  return 0;
  }
 
  2)
  #includestdio.h
  int main()
  {
  extern int a;
  printf(%d,sizeof(a));
  return 0;
  }
 
 
 
  --
  HARSHIT PAHUJA
  M.N.N.I.T.
  ALLAHABAD
 
  --
  You received this message because you are subscribed to the Google
 Groups
  Algorithm Geeks group.
  To post to this group, send email to algogeeks@googlegroups.com.
  To unsubscribe from this group, send email to
  algogeeks+unsubscr...@googlegroups.com.
  For more options, visit this group at
  http://groups.google.com/group/algogeeks?hl=en.
 
 


 --
 *Piyush Sinha*
 *IIIT, Allahabad*
 *+91-8792136657*
 *+91-7483122727*
 *https://www.facebook.com/profile.php?id=10655377926 *

 --
 You received this message because you are subscribed to the Google
 Groups Algorithm Geeks group.
 To post to this group, send email to algogeeks@googlegroups.com.
 To unsubscribe from this group, send email to
 algogeeks+unsubscr...@googlegroups.com.
 For more options, visit this group at
 http://groups.google.com/group/algogeeks?hl=en.




 --
  HARSHIT PAHUJA
 M.N.N.I.T.
 ALLAHABAD





 --
 HARSHIT PAHUJA
 M.N.N.I.T.
 ALLAHABAD


   --
 You received this message because you are subscribed to the Google Groups
 Algorithm Geeks group.
 To post to this group, send email to algogeeks@googlegroups.com.
 To unsubscribe from this group, send email to
 algogeeks+unsubscr...@googlegroups.com.
 For more options, visit this group at
 http://groups.google.com/group/algogeeks?hl=en.


  --
 You received this message because you are subscribed to the Google Groups
 Algorithm Geeks group.
 To post to this group, send email to algogeeks@googlegroups.com.
 To unsubscribe from this group, send email to
 algogeeks+unsubscr...@googlegroups.com.
 For more options, visit this group at
 http://groups.google.com/group/algogeeks?hl=en.




-- 
HARSHIT PAHUJA
M.N.N.I.T.
ALLAHABAD

-- 
You received this message because you are subscribed to the Google Groups 
Algorithm Geeks group.
To post to this group, send email to algogeeks@googlegroups.com.
To unsubscribe from this group, send email to 
algogeeks+unsubscr...@googlegroups.com.
For more options, visit this group at 
http://groups.google.com/group/algogeeks?hl=en.



Re: [algogeeks] Re: Taking Input

2011-07-02 Thread Avi Dullu
#include stdio.h
#include string.h
#include stdlib.h

int main() {
  int n, num, i;
  scanf(%d, n);
  for (i = 0; i  n; ++i) {
char ch = '#';
printf(scanned from %d line: , i + 1);
while (ch != '\n') {
  scanf(%d%c, num, ch);
  printf(%d , num);
}
printf(\n);
  }
  return 0;
}

Better?

Veni Vedi Slumber !


On Sun, Jul 3, 2011 at 2:47 AM, anonymous procrastination 
opamp1...@gmail.com wrote:

 Hey Thanks,,
 I got it.. when I press enter after entering the number it takes that
 as an empty string.

 --
 You received this message because you are subscribed to the Google Groups
 Algorithm Geeks group.
 To post to this group, send email to algogeeks@googlegroups.com.
 To unsubscribe from this group, send email to
 algogeeks+unsubscr...@googlegroups.com.
 For more options, visit this group at
 http://groups.google.com/group/algogeeks?hl=en.



-- 
You received this message because you are subscribed to the Google Groups 
Algorithm Geeks group.
To post to this group, send email to algogeeks@googlegroups.com.
To unsubscribe from this group, send email to 
algogeeks+unsubscr...@googlegroups.com.
For more options, visit this group at 
http://groups.google.com/group/algogeeks?hl=en.



Re: [algogeeks] null macro

2011-07-02 Thread Avi Dullu
#include cstdio

#define NULL1 (void *)0
#define NULL2 0

int main() {
  int* p = NULL1;  // void* being assigned to a int*, C++ complains.
  int* q = NULL2;
  return 0;
}

Veni Vedi Slumber !


On Sat, Jul 2, 2011 at 10:28 PM, Decipher ankurseth...@gmail.com wrote:

 Hey I was asked this question in Broadcom's interview. I thought its worth
 to share it here since its related to null pointer  :

 What's the dif b/w #define NULL (void*)0 and #define NULL 0 ?

  --
 You received this message because you are subscribed to the Google Groups
 Algorithm Geeks group.
 To view this discussion on the web visit
 https://groups.google.com/d/msg/algogeeks/-/mtyvsP4WoGIJ.

 To post to this group, send email to algogeeks@googlegroups.com.
 To unsubscribe from this group, send email to
 algogeeks+unsubscr...@googlegroups.com.
 For more options, visit this group at
 http://groups.google.com/group/algogeeks?hl=en.


-- 
You received this message because you are subscribed to the Google Groups 
Algorithm Geeks group.
To post to this group, send email to algogeeks@googlegroups.com.
To unsubscribe from this group, send email to 
algogeeks+unsubscr...@googlegroups.com.
For more options, visit this group at 
http://groups.google.com/group/algogeeks?hl=en.



[algogeeks] Re: ReadyForZero Programming Challenge

2011-07-02 Thread Tundebabzy
Hi VJ,
You were right. By assuming that the traversal was that of a complete
binary tree with height 9 (not 10), the solution was trivial. Thanks a
lot men

On Jun 23, 6:08 am, VJ vikas.jethn...@gmail.com wrote:
 what if we assume that it's a complete binary tree with height
 10(2^10-1 = 1023) ??

 On Jun 22, 3:05 pm, Tundebabzy tundeba...@gmail.com wrote:







  Thanks so much Wladmir. This algorithm will assume that the second
  character in the string will be the root of the binary tree which
  could be misleading. The root might be the last character in the
  string. The only thing that seems to be universally correct is that
  the first character in the string is a leaf in the binary tree.

  I thought of extracting all the numbers in the string. If i remember
  right, they are 136 in number. Since we are talking of the sum, it
  means that we don't have to bother about which side of the tree the
  digits in the leaves are since 2L + 3R = 5 and 3L + 2R = 5. But then
  again, I got to the question begging for an answer, which exactly in
  these numbers are in the leaves of the binary tree. I could brute
  force a calculation starting by assuming that all the digits are in
  the leaves and then eliminating some of the numbers but that will
  hardly be elegant and could just be called plain cheating even though
  the site does not penalize for getting an answer wrong.

  On Jun 22, 2:49 am, Wladimir Tavares wladimir...@gmail.com wrote:

   I thought of a brute force approach but did not work.

   For each character, I tested two possibilities:
   1. Being a leaf (the leftmost node) and the next character  is the root 
   of a
   subtree.
   2. Be the root of a subtree.

   After testing all possible sums!

   Wladimir Araujo Tavares
   *Federal University of Ceará

   *

   On Tue, Jun 21, 2011 at 10:55 AM, Tundebabzy tundeba...@gmail.com wrote:
Hi guys,
Can anyone help me solve this (I mean explain to me how to solve it):

The following text is an in-order traversal of a binary tree with 1023
nodes. What is the sum of the digits appearing in the leaves?

bDq4i3eFNmjh2oMgjNsIFkRWsonRl=tz9kf8gOpED2gVQrfx49GjR9/
QNqTEJkSzM30p4RfneDc7EmdusIYdxxZ8KKdND==YpLlmN/FkErS7uqVHYIyGEBIhIRX
+mbg6FjVGqfYjobX3F1lSmPpLXXxhux1lV=EzIgGct9pd=ogzAdJU4/
yZOj9=njfvbSo11bcE/yUDHg/J=6DAtmWt+P/VDvE4GyYHlKVGUoksn
+Lzm6Y1VVF6qw=onnKZS=CQQUPeMlUGtMobJEeyIyGBMFj5kHml1waI97qgVt/
yeKWSXxhJyK+As0M0UR2KL+U7klKMlWlxgbIJ2beN0gaMHbIClKrBuroo

+L73CwnbFKRiyKOx8Ke=PPukm1BWYd4Do5nikrT0dgPWlmItCzyZrEiOBgBtEB9FG1qdQJud182
 qYU4Xwry=bR7R5PH=Npab7JQ7gY=MYHW24iw=m2+XayIachr=QVt4giclLluUEu3Dx/
5y7R0GlINqoJc9O6QwEP6a9PRsdt86nZYQUF/
b729lzilkM1PB8mDPACTOGobXSJJQshRQCoJoC9TX6ERou/

tlMWNQTaaU0YmzIFh8t72lgOeIMu4W2S9V8Y5jO8T47ZPRGxeQDWRQlpx1tc3AC4+x2zJxZaYV6
 d4+rxxWgYnYC1ZwUvXNF2YLaRw7ldRXCz9/7xZ3mKBj4Ox51L3PqGXAIB

+OI4pgkUc1X52tOPdRPgpsfi8wNpynfylwjITvNKq93iViQiyiEVGBgazbHQ7boh5tejStBE1SC
 k1fMe1=ycNoxJzhAH4x/
YlQSUOz/2qkl+JxFBt8sqRYXZjZ6R=Gh87ZwS4B0Meo56/WF+40bf2mTgG05KQ/
qR89NMyttHdwwnzvkZuXGrlt/yuStJaXJNkqQ5atDPWqLJy4Q9iylEdI3qmTe6Wmou/
umQG7Q

+aiOXorJde3Z4CzMyTYnCWvudFDZvgEZsuKRiSOurstnxFRlqVQ0LC7WRuS=qCsDcxoDT5dl9NH
 usV4kZ/

I came across this @www.readyforzero.com/challenge(problem2)

Please help me, I'm already tearing my hair out and i'm already
getting bald. I'm stuck because I don't know how it is possible to
figure out a tree from just its in-order traversal

--
You received this message because you are subscribed to the Google 
Groups
Algorithm Geeks group.
To post to this group, send email to algogeeks@googlegroups.com.
To unsubscribe from this group, send email to
algogeeks+unsubscr...@googlegroups.com.
For more options, visit this group at
   http://groups.google.com/group/algogeeks?hl=en.

-- 
You received this message because you are subscribed to the Google Groups 
Algorithm Geeks group.
To post to this group, send email to algogeeks@googlegroups.com.
To unsubscribe from this group, send email to 
algogeeks+unsubscr...@googlegroups.com.
For more options, visit this group at 
http://groups.google.com/group/algogeeks?hl=en.



Re: [algogeeks] Re: help..

2011-07-02 Thread Avi Dullu
Another alternative soln.

int rec_cut(int l, int k) {
  if (l == k) return 0;
  int tmp = k - (l1);
  return 1 + rec_cut(l1, tmp = 0 ? k : tmp);
}

int main() {
  int l, k;
  scanf(%d%d, l, k);
  printf(%d\n, rec_cut(l, k));
  return 0;
}

Veni Vedi Slumber !


On Sat, Jul 2, 2011 at 9:47 PM, varun pahwa varunpahwa2...@gmail.comwrote:

 @sunny thnx for the correction.


 On Sat, Jul 2, 2011 at 9:16 AM, varun pahwa varunpahwa2...@gmail.comwrote:

 @sunny ya  i wanted to write the while(k % m == 0)


 On Sat, Jul 2, 2011 at 3:47 AM, sameer.mut...@gmail.com 
 sameer.mut...@gmail.com wrote:

 nn-1  is the expression to find out if n is a power of 2...If nn-1
 returns 0 its a power of 2 else its not.
 And what sunny said is also ryt


 On Sat, Jul 2, 2011 at 3:47 PM, sunny agrawal 
 sunny816.i...@gmail.comwrote:

 @cegprakash
 Expression resets the least significant set bit


  On Sat, Jul 2, 2011 at 3:20 PM, mohit goel 
 mohitgoel291...@gmail.comwrote:

 May be this can work.give any counter example...
 int count;
 main()
 {
   int l,rope,cuts;
   scanf(%d%d,l,rope);
   count =0;

find_cuts(l,rope);
printf(cuts needed is %d,count);
getch();
return 0;
}

  int find_cuts(int l,int rope)

  {

 if(l==rope)
 return count;
  count++;
  printf(%d,count);
  l=l/2;
  if(l==rope)
  return count;
  if(ropel)
  rope =rope-l;

  find_cuts(l,rope);


  }

 --
 You received this message because you are subscribed to the Google
 Groups Algorithm Geeks group.
 To post to this group, send email to algogeeks@googlegroups.com.
 To unsubscribe from this group, send email to
 algogeeks+unsubscr...@googlegroups.com.
 For more options, visit this group at
 http://groups.google.com/group/algogeeks?hl=en.




 --
 Sunny Aggrawal
 B-Tech IV year,CSI
 Indian Institute Of Technology,Roorkee

  --
 You received this message because you are subscribed to the Google
 Groups Algorithm Geeks group.
 To post to this group, send email to algogeeks@googlegroups.com.
 To unsubscribe from this group, send email to
 algogeeks+unsubscr...@googlegroups.com.
 For more options, visit this group at
 http://groups.google.com/group/algogeeks?hl=en.


  --
 You received this message because you are subscribed to the Google Groups
 Algorithm Geeks group.
 To post to this group, send email to algogeeks@googlegroups.com.
 To unsubscribe from this group, send email to
 algogeeks+unsubscr...@googlegroups.com.
 For more options, visit this group at
 http://groups.google.com/group/algogeeks?hl=en.




 --
 Varun Pahwa
 B.Tech (IT)
 7th Sem.
 Indian Institute of Information Technology Allahabad.
 Ph : 09793899112 ,08011820777
 Official Email :: rit2008...@iiita.ac.in
 Another Email :: varunpahwa.ii...@gmail.com

 People who fail to plan are those who plan to fail.




 --
 Varun Pahwa
 B.Tech (IT)
 7th Sem.
 Indian Institute of Information Technology Allahabad.
 Ph : 09793899112 ,08011820777
 Official Email :: rit2008...@iiita.ac.in
 Another Email :: varunpahwa.ii...@gmail.com

 People who fail to plan are those who plan to fail.

  --
 You received this message because you are subscribed to the Google Groups
 Algorithm Geeks group.
 To post to this group, send email to algogeeks@googlegroups.com.
 To unsubscribe from this group, send email to
 algogeeks+unsubscr...@googlegroups.com.
 For more options, visit this group at
 http://groups.google.com/group/algogeeks?hl=en.


-- 
You received this message because you are subscribed to the Google Groups 
Algorithm Geeks group.
To post to this group, send email to algogeeks@googlegroups.com.
To unsubscribe from this group, send email to 
algogeeks+unsubscr...@googlegroups.com.
For more options, visit this group at 
http://groups.google.com/group/algogeeks?hl=en.



[algogeeks] Re: Sort - Consecutive Array in O(n)

2011-07-02 Thread Gene
You can use a count sort, but you need an array to store the counts.
The oppilas algorithm needs only constant extra storage.

On Jun 30, 7:23 am, hary rathor harry.rat...@gmail.com wrote:
 can we not use count sort because of O(n) .?

-- 
You received this message because you are subscribed to the Google Groups 
Algorithm Geeks group.
To post to this group, send email to algogeeks@googlegroups.com.
To unsubscribe from this group, send email to 
algogeeks+unsubscr...@googlegroups.com.
For more options, visit this group at 
http://groups.google.com/group/algogeeks?hl=en.



Re: [algogeeks] Re: Interview Question

2011-07-02 Thread varun pahwa
@aditya. xor all elements mean that. take xor of each element of 1st array
store in a variable that take xor of variable and each element of the second
array if all elements are common then the variable will be 0 some where.
var = a[0];
for(i = 1; i  sizeof(a)/sizeof(a[0]); i++)
var = var ^ a[i];
for(i = 0; i  sizeof(b)/sizeof(b[0]); i++)
var = var ^ b[i];


On Sat, Jul 2, 2011 at 2:19 PM, aditya kumar
aditya.kumar130...@gmail.comwrote:

 @mohit..:i dint get the logic behind XOR plz explain ..nd ya i dont think
 dat you can find second largest in less than O(n).


 On Sun, Jul 3, 2011 at 2:43 AM, mohit mittal mohitm.1...@gmail.comwrote:

 Dont think that the corresponding elements should be same.
 XOR Should do it anyway.

 Btw other question How would you find the second largest element in an
 array using minimum no of comparisons?Any thing better than O(n).?


 On Sun, Jul 3, 2011 at 2:41 AM, aditya kumar 
 aditya.kumar130...@gmail.com wrote:

 xor will only result if corresponding elements are same . what if in both
 the array set of integers are same but they arnt corresponding to each other
 ??


 On Sun, Jul 3, 2011 at 2:37 AM, Dumanshu duman...@gmail.com wrote:

 xor all the elements of both arrays ==0
 sum of 1st array == sum of 2nd array
 no. of elements in 1st == no. of elements in 2nd
 if the above conditions are met, they have the same set.
 m i missin sth?
 On Jul 3, 1:23 am, mittal mohitm.1...@gmail.com wrote:
  Given two arrays of numbers, find if each of the two arrays have the
 same
  set of ntegers ? Suggest an algo which can run faster than NlogN
 without
  extra space?

 --
 You received this message because you are subscribed to the Google
 Groups Algorithm Geeks group.
 To post to this group, send email to algogeeks@googlegroups.com.
 To unsubscribe from this group, send email to
 algogeeks+unsubscr...@googlegroups.com.
 For more options, visit this group at
 http://groups.google.com/group/algogeeks?hl=en.


  --
 You received this message because you are subscribed to the Google Groups
 Algorithm Geeks group.
 To post to this group, send email to algogeeks@googlegroups.com.
 To unsubscribe from this group, send email to
 algogeeks+unsubscr...@googlegroups.com.
 For more options, visit this group at
 http://groups.google.com/group/algogeeks?hl=en.




 --
 Mohit Mittal
 4th year , Computer Engineering
 Student-Coordinator , DTU WebTeam
 Delhi Technological University

  --
 You received this message because you are subscribed to the Google Groups
 Algorithm Geeks group.
 To post to this group, send email to algogeeks@googlegroups.com.
 To unsubscribe from this group, send email to
 algogeeks+unsubscr...@googlegroups.com.
 For more options, visit this group at
 http://groups.google.com/group/algogeeks?hl=en.


  --
 You received this message because you are subscribed to the Google Groups
 Algorithm Geeks group.
 To post to this group, send email to algogeeks@googlegroups.com.
 To unsubscribe from this group, send email to
 algogeeks+unsubscr...@googlegroups.com.
 For more options, visit this group at
 http://groups.google.com/group/algogeeks?hl=en.




-- 
Varun Pahwa
B.Tech (IT)
7th Sem.
Indian Institute of Information Technology Allahabad.
Ph : 09793899112 ,08011820777
Official Email :: rit2008...@iiita.ac.in
Another Email :: varunpahwa.ii...@gmail.com

People who fail to plan are those who plan to fail.

-- 
You received this message because you are subscribed to the Google Groups 
Algorithm Geeks group.
To post to this group, send email to algogeeks@googlegroups.com.
To unsubscribe from this group, send email to 
algogeeks+unsubscr...@googlegroups.com.
For more options, visit this group at 
http://groups.google.com/group/algogeeks?hl=en.



[algogeeks] Re: Number of Comparisons!

2011-07-02 Thread Dave
@Udit: This can't be correct. For n = 3, it predicts 4-1/2
comparisons. I don't know what half of a comparison is. Three
comparisons are all that is required. In fact, for all odd n, it
predicts a non-integer number of comparisons.

Dave

On Jul 2, 3:51 pm, udit sharma sharmaudit...@gmail.com wrote:
 If n is odd, then we perform
 3*n/2 comparisons. If n is even, we perform 1 + 3*(n-2)/2

 --
 Regards
  UDIT
  DU- MCA

-- 
You received this message because you are subscribed to the Google Groups 
Algorithm Geeks group.
To post to this group, send email to algogeeks@googlegroups.com.
To unsubscribe from this group, send email to 
algogeeks+unsubscr...@googlegroups.com.
For more options, visit this group at 
http://groups.google.com/group/algogeeks?hl=en.



Re: [algogeeks] Re: Number of Comparisons!

2011-07-02 Thread DIPANKAR DUTTA
plz refer sahani's book of algo..:)

On 7/2/11, Dave dave_and_da...@juno.com wrote:
 @Udit: This can't be correct. For n = 3, it predicts 4-1/2
 comparisons. I don't know what half of a comparison is. Three
 comparisons are all that is required. In fact, for all odd n, it
 predicts a non-integer number of comparisons.

 Dave

 On Jul 2, 3:51 pm, udit sharma sharmaudit...@gmail.com wrote:
 If n is odd, then we perform
 3*n/2 comparisons. If n is even, we perform 1 + 3*(n-2)/2

 --
 Regards
  UDIT
  DU- MCA

 --
 You received this message because you are subscribed to the Google Groups
 Algorithm Geeks group.
 To post to this group, send email to algogeeks@googlegroups.com.
 To unsubscribe from this group, send email to
 algogeeks+unsubscr...@googlegroups.com.
 For more options, visit this group at
 http://groups.google.com/group/algogeeks?hl=en.




-- 
Thanks and Regards,
--
**DIPANKAR DUTTA
M-TECH,Computer Science  Engg.
EC Dept,IIT ROORKEE
Uttarakhand , India – 247667
---
website:http://people.iitr.ernet.in/shp/09535009/Website/index.html
ph no-09045809987
Lab: 286454
email:dipan...@iitr.ernet.in

-- 
You received this message because you are subscribed to the Google Groups 
Algorithm Geeks group.
To post to this group, send email to algogeeks@googlegroups.com.
To unsubscribe from this group, send email to 
algogeeks+unsubscr...@googlegroups.com.
For more options, visit this group at 
http://groups.google.com/group/algogeeks?hl=en.



Re: [algogeeks] output plzzzz

2011-07-02 Thread Sandeep Jain
One thing to remember is that, *sizeof* operator does not evaluate the
expression used as the parameter, it only evaluates type of the parameter.
So, in case 2, sizeof(a) == sizeof(int) == 4.
We never actually access 'a'



Regards,
Sandeep Jain
Member of Technical Staff, Adobe Systems, India




On Sun, Jul 3, 2011 at 3:13 AM, HARSH PAHUJA hpahuja.mn...@gmail.comwrote:

 thanx i got it :)


 On Sat, Jul 2, 2011 at 1:57 PM, santosh mahto santoshbit2...@gmail.comwrote:

 this will help

 -sizeof operator only need declaration not definition.since sizeof
 operator calculates size at
 compile time.
 At compile time code doesnot seek for definition.it only need
 declaration.if any variable is declared with extern,it thinks that
 its definition will be found at linking time.

 In ur question:
 1 case. when a =20 is done compiler see that its value is declared.so its
 OK .but at link time since def of a is not found
 so linker error. since linker tries to find the actual memory address
 where the def of a is.

 see after compilation the symbol table for a will be as

 symbolname| memory location| value
  a |find later(garbage)| 20

 so there is no problem with compiler .but at link time  garbage value need
 to be filled with actual memory address
 since def is not there so linker error.

 case2. there will be noentry in the symbol table for a. as code doesnot
 include any defintion related operation.


 iF anything helps you, dont forget to say thanks

 Thanks
 Santosh





 On Sun, Jul 3, 2011 at 1:38 AM, HARSH PAHUJA hpahuja.mn...@gmail.comwrote:

 will ny1 xplain this extern concept
 y this is a linker error?


 On Sat, Jul 2, 2011 at 12:49 PM, HARSH PAHUJA 
 hpahuja.mn...@gmail.comwrote:

 @piyush  plz explain how are u getting this..


 On Sat, Jul 2, 2011 at 12:10 PM, Piyush Sinha ecstasy.piy...@gmail.com
  wrote:

 i)  [Linker error] undefined reference to `a'

 ii) 4

 On 7/3/11, HARSH PAHUJA hpahuja.mn...@gmail.com wrote:
  1)
  #includestdio.h
  int main()
  {
  extern int a;
  a=20;
  printf(%d,sizeof(a));
  return 0;
  }
 
  2)
  #includestdio.h
  int main()
  {
  extern int a;
  printf(%d,sizeof(a));
  return 0;
  }
 
 
 
  --
  HARSHIT PAHUJA
  M.N.N.I.T.
  ALLAHABAD
 
  --
  You received this message because you are subscribed to the Google
 Groups
  Algorithm Geeks group.
  To post to this group, send email to algogeeks@googlegroups.com.
  To unsubscribe from this group, send email to
  algogeeks+unsubscr...@googlegroups.com.
  For more options, visit this group at
  http://groups.google.com/group/algogeeks?hl=en.
 
 


 --
 *Piyush Sinha*
 *IIIT, Allahabad*
 *+91-8792136657*
 *+91-7483122727*
 *https://www.facebook.com/profile.php?id=10655377926 *

 --
 You received this message because you are subscribed to the Google
 Groups Algorithm Geeks group.
 To post to this group, send email to algogeeks@googlegroups.com.
 To unsubscribe from this group, send email to
 algogeeks+unsubscr...@googlegroups.com.
 For more options, visit this group at
 http://groups.google.com/group/algogeeks?hl=en.




 --
  HARSHIT PAHUJA
 M.N.N.I.T.
 ALLAHABAD





 --
 HARSHIT PAHUJA
 M.N.N.I.T.
 ALLAHABAD


   --
 You received this message because you are subscribed to the Google Groups
 Algorithm Geeks group.
 To post to this group, send email to algogeeks@googlegroups.com.
 To unsubscribe from this group, send email to
 algogeeks+unsubscr...@googlegroups.com.
 For more options, visit this group at
 http://groups.google.com/group/algogeeks?hl=en.


  --
 You received this message because you are subscribed to the Google Groups
 Algorithm Geeks group.
 To post to this group, send email to algogeeks@googlegroups.com.
 To unsubscribe from this group, send email to
 algogeeks+unsubscr...@googlegroups.com.
 For more options, visit this group at
 http://groups.google.com/group/algogeeks?hl=en.




 --
 HARSHIT PAHUJA
 M.N.N.I.T.
 ALLAHABAD


  --
 You received this message because you are subscribed to the Google Groups
 Algorithm Geeks group.
 To post to this group, send email to algogeeks@googlegroups.com.
 To unsubscribe from this group, send email to
 algogeeks+unsubscr...@googlegroups.com.
 For more options, visit this group at
 http://groups.google.com/group/algogeeks?hl=en.


-- 
You received this message because you are subscribed to the Google Groups 
Algorithm Geeks group.
To post to this group, send email to algogeeks@googlegroups.com.
To unsubscribe from this group, send email to 
algogeeks+unsubscr...@googlegroups.com.
For more options, visit this group at 
http://groups.google.com/group/algogeeks?hl=en.



[algogeeks] Dynamic Programming problem

2011-07-02 Thread Edmon
I need a help with this dynamic programming problem please.
It is from the entrance exam practice problem set:

Given an integer sequence x_1 ... x_n is there a nonempty sub
sequences
which sums to zero?

Describe - no code necessary - a dynamic programming solution based on
the predicate:

A nonempty sub sequence of x_1 ... x_n has sum s

I think that solution should be built on the
recursive backtracking function that selects a min
 from these subsequences choices, but I think I am missing lots of
details
here.

Any assistance is appreciated. Thank you in advance.

-- 
You received this message because you are subscribed to the Google Groups 
Algorithm Geeks group.
To post to this group, send email to algogeeks@googlegroups.com.
To unsubscribe from this group, send email to 
algogeeks+unsubscr...@googlegroups.com.
For more options, visit this group at 
http://groups.google.com/group/algogeeks?hl=en.



Re: [algogeeks] Dynamic Programming problem

2011-07-02 Thread Navneet Gupta
This is a recently discussed problem in this group.

Refer to subset sum problem. http://en.wikipedia.org/wiki/Subset_sum_problem

On Sun, Jul 3, 2011 at 6:34 AM, Edmon ebeg...@gmail.com wrote:
 I need a help with this dynamic programming problem please.
 It is from the entrance exam practice problem set:

 Given an integer sequence x_1 ... x_n is there a nonempty sub
 sequences
 which sums to zero?

 Describe - no code necessary - a dynamic programming solution based on
 the predicate:

 A nonempty sub sequence of x_1 ... x_n has sum s

 I think that solution should be built on the
 recursive backtracking function that selects a min
  from these subsequences choices, but I think I am missing lots of
 details
 here.

 Any assistance is appreciated. Thank you in advance.

 --
 You received this message because you are subscribed to the Google Groups 
 Algorithm Geeks group.
 To post to this group, send email to algogeeks@googlegroups.com.
 To unsubscribe from this group, send email to 
 algogeeks+unsubscr...@googlegroups.com.
 For more options, visit this group at 
 http://groups.google.com/group/algogeeks?hl=en.





-- 
--Navneet

-- 
You received this message because you are subscribed to the Google Groups 
Algorithm Geeks group.
To post to this group, send email to algogeeks@googlegroups.com.
To unsubscribe from this group, send email to 
algogeeks+unsubscr...@googlegroups.com.
For more options, visit this group at 
http://groups.google.com/group/algogeeks?hl=en.



Re: [algogeeks] Re: Interview Question

2011-07-02 Thread Pranav Agarwal
I think that the above algo will fail for the following two arrays:
a={2,2,3,3}
b={4,4,1,1}

sum(a)=sum(b);
a^b=0;
len(a)=len(b);

Correct me if i am wrong!

Pranav

On Sun, Jul 3, 2011 at 7:43 AM, varun pahwa varunpahwa2...@gmail.comwrote:

 @aditya. xor all elements mean that. take xor of each element of 1st array
 store in a variable that take xor of variable and each element of the second
 array if all elements are common then the variable will be 0 some where.
 var = a[0];
 for(i = 1; i  sizeof(a)/sizeof(a[0]); i++)
 var = var ^ a[i];
 for(i = 0; i  sizeof(b)/sizeof(b[0]); i++)
 var = var ^ b[i];



 On Sat, Jul 2, 2011 at 2:19 PM, aditya kumar aditya.kumar130...@gmail.com
  wrote:

 @mohit..:i dint get the logic behind XOR plz explain ..nd ya i dont think
 dat you can find second largest in less than O(n).


 On Sun, Jul 3, 2011 at 2:43 AM, mohit mittal mohitm.1...@gmail.comwrote:

 Dont think that the corresponding elements should be same.
 XOR Should do it anyway.

 Btw other question How would you find the second largest element in an
 array using minimum no of comparisons?Any thing better than O(n).?


 On Sun, Jul 3, 2011 at 2:41 AM, aditya kumar 
 aditya.kumar130...@gmail.com wrote:

 xor will only result if corresponding elements are same . what if in
 both the array set of integers are same but they arnt corresponding to each
 other ??


 On Sun, Jul 3, 2011 at 2:37 AM, Dumanshu duman...@gmail.com wrote:

 xor all the elements of both arrays ==0
 sum of 1st array == sum of 2nd array
 no. of elements in 1st == no. of elements in 2nd
 if the above conditions are met, they have the same set.
 m i missin sth?
 On Jul 3, 1:23 am, mittal mohitm.1...@gmail.com wrote:
  Given two arrays of numbers, find if each of the two arrays have the
 same
  set of ntegers ? Suggest an algo which can run faster than NlogN
 without
  extra space?

 --
 You received this message because you are subscribed to the Google
 Groups Algorithm Geeks group.
 To post to this group, send email to algogeeks@googlegroups.com.
 To unsubscribe from this group, send email to
 algogeeks+unsubscr...@googlegroups.com.
 For more options, visit this group at
 http://groups.google.com/group/algogeeks?hl=en.


  --
 You received this message because you are subscribed to the Google
 Groups Algorithm Geeks group.
 To post to this group, send email to algogeeks@googlegroups.com.
 To unsubscribe from this group, send email to
 algogeeks+unsubscr...@googlegroups.com.
 For more options, visit this group at
 http://groups.google.com/group/algogeeks?hl=en.




 --
 Mohit Mittal
 4th year , Computer Engineering
 Student-Coordinator , DTU WebTeam
 Delhi Technological University

  --
 You received this message because you are subscribed to the Google Groups
 Algorithm Geeks group.
 To post to this group, send email to algogeeks@googlegroups.com.
 To unsubscribe from this group, send email to
 algogeeks+unsubscr...@googlegroups.com.
 For more options, visit this group at
 http://groups.google.com/group/algogeeks?hl=en.


  --
 You received this message because you are subscribed to the Google Groups
 Algorithm Geeks group.
 To post to this group, send email to algogeeks@googlegroups.com.
 To unsubscribe from this group, send email to
 algogeeks+unsubscr...@googlegroups.com.
 For more options, visit this group at
 http://groups.google.com/group/algogeeks?hl=en.




 --
 Varun Pahwa
 B.Tech (IT)
 7th Sem.
 Indian Institute of Information Technology Allahabad.
 Ph : 09793899112 ,08011820777
 Official Email :: rit2008...@iiita.ac.in
 Another Email :: varunpahwa.ii...@gmail.com

 People who fail to plan are those who plan to fail.

  --
 You received this message because you are subscribed to the Google Groups
 Algorithm Geeks group.
 To post to this group, send email to algogeeks@googlegroups.com.
 To unsubscribe from this group, send email to
 algogeeks+unsubscr...@googlegroups.com.
 For more options, visit this group at
 http://groups.google.com/group/algogeeks?hl=en.


-- 
You received this message because you are subscribed to the Google Groups 
Algorithm Geeks group.
To post to this group, send email to algogeeks@googlegroups.com.
To unsubscribe from this group, send email to 
algogeeks+unsubscr...@googlegroups.com.
For more options, visit this group at 
http://groups.google.com/group/algogeeks?hl=en.